Fecal Incontinence and Constipation in Children: Case Studies [1 ed.] 9780367151614, 9780367151805, 9780429055522, 9780429619243, 9780429617096, 9780429621390

This book focuses on the management of children with fecal incontinence and constipation. Despite accurate anatomic reco

536 26 18MB

Pages [203] Year 2019

Report DMCA / Copyright

DOWNLOAD FILE

Polecaj historie

Fecal Incontinence and Constipation in Children: Case Studies [1 ed.]
 9780367151614, 9780367151805, 9780429055522, 9780429619243, 9780429617096, 9780429621390

Table of contents :

PART I: BOWEL MANAGEMENT. General guidelines for bowel management. Bowel management program setup: The basics and long-term follow-up. PART II: ANORECTAL MALFORMATIONS. A patient with good surgical anatomy after anorectal malformation (ARM). repair with good potential for bowel control. A patient with good surgical anatomy following anorectal malformation (ARM). repair with fair potential for bowel control. A patient with a well done anatomic anorectal malformation (ARM) repair, but with poor potential for bowel control. A patient with a history of a cloacal malformation who needs colorectal, urological, and gynecological collaboration. A young adult with prior surgery for anorectal malformation (ARM) with fecal incontinence. A patient with an anorectal malformation (ARM) with fecal incontinence. who is a candidate for a sacral nerve stimulator (SNS). PART III: HIRSCHSPRUNG DISEASE. A patient with good surgical anatomy and hypomotility after a Hirschsprung pull-through. A patient with good surgical anatomy and hypermotility after a redo pull-through. for Hirschsprung disease. An older child with Hirschsprung disease (HD) and hypomotility. A patient with total colonic Hirschsprung disease and soiling. A teenager with prior surgery for Hirschsprung disease who has constipation. PART IV: SPINAL ANOMALIES. A patient with a hypodeveloped sacrum and fecal and urinary incontinence. A patient with a spinal anomaly and fecal incontinence. A pediatric patient with spina bifida in need of a urological reconstruction. A young adult with quadriplegia and fecal incontinence due to spinal cord injury (SCI). PART V: INTRODUCTION TO FUNCTIONAL CONSTIPATION. A case of diffuse colonic dysmotility. A patient with chronic constipation and sphincter dysfunction. A patient with severe functional constipation, fecal impaction, and no soiling. A patient with severe functional constipation, fecal impaction, and soiling. Success with a rectal enema regimen, but now unable to tolerate rectal administration. A patient with severe functional constipation who has failed laxative treatment. and both antegrade and rectal enemas. A patient who has recurrent constipation and soiling following colonic resection. A young adult with intractable constipation and diffuse colonic dysmotility. An adult with pelvic floor dyssynergia. A patient with severe constipation and a behavioral disorder. An adult with incontinence after a low anterior resection. Two adults with incontinence after childbirth. A young adult with rectal pain and fecal urgency who is a candidate for sacral. nerve stimulation. An adult with soiling following an ileoanal pouch. PART VI: RADIOLOGY. Which X-ray is worse?. Interesting radiological findings. PART VII: MYTHS. Colorectal surgical myths. PART VIII: MEDICATION PROTOCOLS. Medication protocols. Index.

Citation preview

Fecal Incontinence and Constipation in Children Case Studies

Fecal Incontinence and Constipation in Children Case Studies

Edited by

Onnalisa Nash, MS, CPNP

Center for Colorectal and Pelvic Reconstruction Surgery Nationwide Children’s Hospital Columbus, Ohio, USA

Julie M. Choueiki, MSN, RN, CPEN

Center for Colorectal and Pelvic Reconstruction Surgery Nationwide Children’s Hospital Columbus, Ohio, USA

Marc A. Levitt, MD

Center for Colorectal and Pelvic Reconstruction Surgery Nationwide Children’s Hospital The Ohio State University Columbus, Ohio, USA

CRC Press Taylor & Francis Group 6000 Broken Sound Parkway NW, Suite 300 Boca Raton, FL 33487-2742 © 2020 by Taylor & Francis Group, LLC CRC Press is an imprint of Taylor & Francis Group, an Informa business No claim to original U.S. Government works Printed on acid-free paper International Standard Book Number-13: 978-0-367-15161-4 (Paperback) 978-0-367-15180-5 (Hardback) This book contains information obtained from authentic and highly regarded sources. While all reasonable efforts have been made to publish reliable data and information, neither the author[s] nor the publisher can accept any legal responsibility or liability for any errors or omissions that may be made. The publishers wish to make clear that any views or opinions expressed in this book by individual editors, authors or contributors are personal to them and do not necessarily reflect the views/opinions of the publishers. The information or guidance contained in this book is intended for use by medical, scientific or health-care professionals and is provided strictly as a supplement to the medical or other professional’s own judgement, their knowledge of the patient’s medical history, relevant manufacturer’s instructions and the appropriate best practice guidelines. Because of the rapid advances in medical science, any information or advice on dosages, procedures or diagnoses should be independently verified. The reader is strongly urged to consult the relevant national drug formulary and the drug companies’ and device or material manufacturers’ printed instructions, and their websites, before administering or utilizing any of the drugs, devices or materials mentioned in this book. This book does not indicate whether a particular treatment is appropriate or suitable for a particular individual. Ultimately it is the sole responsibility of the medical professional to make his or her own professional judgements, so as to advise and treat patients appropriately. The authors and publishers have also attempted to trace the copyright holders of all material reproduced in this publication and apologize to copyright holders if permission to publish in this form has not been obtained. If any copyright material has not been acknowledged please write and let us know so we may rectify in any future reprint. Except as permitted under U.S. Copyright Law, no part of this book may be reprinted, reproduced, transmitted, or utilized in any form by any electronic, mechanical, or other means, now known or hereafter invented, including photocopying, microfilming, and recording, or in any information storage or retrieval system, without written permission from the publishers. For permission to photocopy or use material electronically from this work, please access www.copyright.com (http://www.copyright. com/) or contact the Copyright Clearance Center, Inc. (CCC), 222 Rosewood Drive, Danvers, MA 01923, 978-750-8400. CCC is a not-forprofit organization that provides licenses and registration for a variety of users. For organizations that have been granted a photocopy license by the CCC, a separate system of payment has been arranged. Trademark Notice: Product or corporate names may be trademarks or registered trademarks, and are used only for identification and explanation without intent to infringe. Visit the Taylor & Francis Web site at http://www.taylorandfrancis.com and the CRC Press Web site at http://www.crcpress.com

Contents

Preface Contributors Acknowledgments

ix xi xiii

PART I  BOWEL MANAGEMENT

1

1 General guidelines for bowel management Stephanie J. Vyrostek 2 Bowel management program setup: The basics and long-term follow-up Stephanie J. Vyrostek and Laura J. Weaver

2

PART II  ANORECTAL MALFORMATIONS 3 A patient with good surgical anatomy after an anorectal malformation (ARM) repair with good potential for bowel control Erin M. Shann 4 A patient with good surgical anatomy following an anorectal malformation (ARM) repair with fair potential for bowel control Erin M. Shann 5 A patient with a good anatomic anorectal malformation (ARM) repair, but with poor potential for bowel control Catherine Trimble 6 A patient with a history of a cloacal malformation who needs colorectal, urological, and gynecological collaboration Kristina Booth 7 A young adult with prior surgery for an anorectal malformation (ARM) with fecal incontinence Onnalisa Nash 8 A patient with an anorectal malformation (ARM) with fecal incontinence who is a candidate for a sacral nerve stimulator (SNS) Catherine Trimble

6 11 13 19 26 30 35 38

PART III  HIRSCHSPRUNG DISEASE

43

9 A patient with good surgical anatomy and hypomotility after a Hirschsprung pull-through Leah Moore 10 A patient with good surgical anatomy and hypermotility after a redo pull-through for Hirschsprung disease Leah Moore 11 A child with Hirschsprung disease (HD) and hypomotility Lindsay Reilly

46 49 53

v

Contents

12 A patient with total colonic Hirschsprung disease and soiling Stacie Leeper 13 A teenager with prior surgery for Hirschsprung disease who has constipation Stacie Leeper

vi

57 62

PART IV  SPINAL ANOMALIES

67

14 A patient with a hypodeveloped sacrum and fecal and urinary incontinence Cassie do Carmo 15 A patient with a spinal anomaly and fecal incontinence Cassie do Carmo 16 A pediatric patient with spina bifida in need of a urological reconstruction Cheryl Baxter 17 A young adult with quadriplegia and fecal incontinence due to spinal cord injury (SCI) Cheryl Baxter

68

PART V  FUNCTIONAL CONSTIPATION

83

18 A case of diffuse colonic dysmotility Sarah Driesbach 19 A patient with chronic constipation and sphincter dysfunction Sarah Driesbach 20 A patient with severe functional constipation, fecal impaction, and no soiling Julie Zipfel and Marc A. Levitt 21 A patient with severe functional constipation, fecal impaction, and soiling Julie Zipfel and Marc A. Levitt 22 A patient with a successful rectal enema regimen but who now is unable to tolerate rectal administration Andrea Wagner 23 A patient with severe functional constipation who has failed laxative treatment and both rectal and antegrade enemas Julie Gerberick 24 A patient who has recurrent constipation and soiling following colonic resection Andrea Wagner 25 A young adult with intractable constipation and diffuse colonic dysmotility Alessandra Gasior and Amber Traugott 26 A young adult with pelvic floor dyssynergia Alessandra Gasior and Amber Traugott 27 A patient with severe constipation and a behavioral disorder Katrina Hall, Charae Keys, and Rose Lucey Schroedl 28 A young adult with incontinence after a low anterior resection Alicia Finn, Scott Lake, and Amber Traugott 29 Two adults with incontinence after childbirth Stephanie Dolan, Amber Traugott, and Pooja Zahora 30 A young adult with rectal pain and fecal urgency who is a candidate for sacral nerve stimulation Alessandra Gasior and Amber Traugott

85

73 76 79

93 98 102 108 112 120 124 126 128 132 135 141

Contents

31 An adult with soiling following an ileoanal pouch Alicia Finn and Amber Traugott

144

PART VI RADIOLOGY

149

32 Which X-ray is worse? Onnalisa Nash 33 Interesting radiological findings Onnalisa Nash and Marc A. Levitt

150

PART VII MYTHS

163

34 Colorectal surgical myths Julie M. Choueiki

164

PART VIII  MEDICATION PROTOCOLS

173

35 Medication protocols Meghan Fisher and Onnalisa Nash

174

Index

183

156

vii

Preface

Patients with anorectal malformations (ARMs), Hirschsprung disease (HD), fecal incontinence from a variety of conditions, and colonic motility disorders often require care from specialists across a variety of fields throughout their lives. These include colorectal surgery, urology, gynecology, and GI motility, as well as orthopedics, neurosurgery, anesthesia, pathology, radiology, psychology, social work, and nutrition, amongst many others. Perhaps most important to their achievement of a good functional result is their connection to superb nursing care. Having met many parents with newborns diagnosed with colorectal problems, I have made several observations. First, no parent ever thinks their child could have a problem with stooling—this is a physiologic ability that is taken for granted. When told this is a problem, they are usually shocked. Second, when discussing with parents that their child will need surgery to correct their colorectal anatomy, they do not focus on the surgical technique and elegance of the anal reconstruction, as I do. Instead, they focus on whether that technique will allow their child to stool without difficulty, and whether school, sleep overs, and overnight camp will be options for them. As surgeons, we need to remember this—we always need to understand what it is that the family wishes for us to deliver to them. As proud of our surgical skills as we are, it is the functional outcome that matters most to our patients. I like to say that a complex colorectal operation takes about 4 hours to perform, but in order to get a good result, it takes an additional 96 hours of work—the vast majority of which involves nursing care. From the very beginning of my journey in the field of pediatric colorectal surgery, which began in 1992 as a budding medical student, the value of a good nursing partner became clear to me. Patients and their families tell nurses things they would never tell their doctors. Nurses have a spirit about them that is unique—the genuine devotion to helping their patient and never seeming to want anything in return other than the patient’s smile. Their skills in identifying problems, solving them, being willing to get down in the weeds, and always striving to fill the gaps are unique to the profession.

We have attempted in this book to capture some of these special moments represented in the illustrated cases you are about to encounter. We strove to help other caregivers understand the daily struggle of improving a patient’s quality of life and to convey to the readers of this book the skills and tricks to achieve good results. I am so convinced, and often shout from the rooftops that, without my nursing partners, I would have achieved very little as a surgeon. It was an honor

ix

Preface

to work on this book with them and to have my name appear as an author beside theirs. We hope you enjoy reading this book and that it helps you help many children. If we have achieved that lofty goal, then we will feel very gratified. I am forever indebted to the model of education my father, a neurologist, taught me using cases to illustrate learning points, which were codified originally in his books Neurology for the House Officer and Case Studies in Neurology for the House Officer. Both of my parents, Eva and Larry Levitt, helped me to learn empathy and the importance of hard work. I also wish to thank Shary, my wife, and my children, Sam, Raquel, and Jess who generously gave me up so I could spend time helping patients; their confidence in me and tireless support are the basis of my success. Marc A. Levitt, MD Note: Case studies in this book are derived from actual patients. Identifying information such as gender or age may have been altered.

x

Contributors

Cheryl Baxter, MSN, CPNP Center for Colorectal and Pelvic Reconstruction Surgery Nationwide Children’s Hospital Columbus, Ohio

Meghan Fisher, BSN, RN Center for Colorectal and Pelvic Reconstruction Surgery Nationwide Children’s Hospital Columbus, Ohio

Kristina Booth, MSN, FNP Center for Colorectal and Pelvic Reconstruction Surgery Nationwide Children’s Hospital Columbus, Ohio

Alessandra Gasior, DO Center for Colorectal and Pelvic Reconstruction Surgery Nationwide Children’s Hospital and Division of Colorectal Surgery The Ohio State University Wexner Medical Center Columbus, Ohio

Julie M. Choueiki, MSN, RN, CPEN Center for Colorectal and Pelvic Reconstruction Surgery Nationwide Children’s Hospital Columbus, Ohio Sara Cramer, BSN, RN Center for Colorectal and Pelvic Reconstruction Surgery Nationwide Children’s Hospital Columbus, Ohio

Julie Gerberick, MS, RN, CEN Center for Colorectal and Pelvic Reconstruction Surgery Nationwide Children’s Hospital Columbus, Ohio

Cassie do Carmo, BSN, RN Center for Colorectal and Pelvic Reconstruction Surgery Nationwide Children’s Hospital Columbus, Ohio

Katrina Hall, MA, CCLS Center for Colorectal and Pelvic Reconstruction Surgery Nationwide Children’s Hospital Columbus, Ohio

Stephanie Dolan, MSN, NP Division of Colorectal Surgery The Ohio State University Wexner Medical Center Columbus, Ohio

Charae Keys, MSW, LISW-S Center for Colorectal and Pelvic Reconstruction Surgery Nationwide Children’s Hospital Columbus, Ohio

Sarah Driesbach, MSN, FNP Center for Colorectal and Pelvic Reconstruction Surgery Nationwide Children’s Hospital Columbus, Ohio

Scott Lake, APRN Division of Colorectal Surgery The Ohio State University Wexner Medical Center Columbus, Ohio

Alicia Finn, APRN Division of Colorectal Surgery The Ohio State University Wexner Medical Center Columbus, Ohio

Stacie Leeper, MSN, CPNP Center for Colorectal and Pelvic Reconstruction Surgery Nationwide Children’s Hospital Columbus, Ohio

xi

Contributors

Marc A. Levitt, MD Center for Colorectal and Pelvic Reconstruction Surgery Nationwide Children’s Hospital and The Ohio State University Columbus, Ohio Leah Moore, BSN, RN Center for Colorectal and Pelvic Reconstruction Surgery Nationwide Children’s Hospital Columbus, Ohio Onnalisa Nash, MS, CPNP Center for Colorectal and Pelvic Reconstruction Surgery Nationwide Children’s Hospital Columbus, Ohio Lindsay Reilly, MSN, FNP Center for Colorectal and Pelvic Reconstruction Surgery Nationwide Children’s Hospital Columbus, Ohio Natalie Rose, BSN, RN Center for Colorectal and Pelvic Reconstruction Surgery Nationwide Children’s Hospital Columbus, Ohio Rose Lucey Schroedl, PhD Center for Colorectal and Pelvic Reconstruction Surgery Nationwide Children’s Hospital Columbus, Ohio Erin M. Shann, BSN, RN Center for Colorectal and Pelvic Reconstruction Surgery Nationwide Children’s Hospital Columbus, Ohio

xii

Amber Traugott, MD Division of Colorectal Surgery The Ohio State University Wexner Medical Center Columbus, Ohio Catherine Trimble, MSN, FNP Center for Colorectal and Pelvic Reconstruction Surgery Nationwide Children’s Hospital Columbus, Ohio Stephanie J. Vyrostek, BSN, RN Center for Colorectal and Pelvic Reconstruction Surgery Nationwide Children’s Hospital Columbus, Ohio Andrea Wagner, MSN, CPNP Center for Colorectal and Pelvic Reconstruction Surgery Nationwide Children’s Hospital Columbus, Ohio Laura J. Weaver, MA Center for Colorectal and Pelvic Reconstruction Surgery Nationwide Children’s Hospital Columbus, Ohio Pooja Zahora, MSN, FNP Center for Colorectal and Pelvic Reconstruction Surgery Nationwide Children’s Hospital Columbus, Ohio Julie Zipfel, BSN, RN Center for Colorectal and Pelvic Reconstruction Surgery Nationwide Children’s Hospital Columbus, Ohio

Acknowledgments

We are grateful to the colleagues listed below for the support they provide in the care of patients, their individual roles in our wonderful and collaborative team, and their help in organizing various aspects of this book. • • • • • • • • • • • • • • • • • • • • • • •

Katrina Abram Brent Adler Hira Ahmad Neetu Bali Jillean Bastian Morris Brown Christina Ching Marissa Condon Jackie Cronau Daniel DaJusta Katherine Deans Karen Diefenbach Carlo DiLorenzo Rob Dyckes Pam Edson Adrienne Flood Molly Fuchs Devin Halleran Geri Hewitt Libbey Hoang Jennie Hoffman Ronda Hutt Venkata R. Jayanthi

• • • • • • • • • • • • • • • • • • • • • • •

Victoria Lane Jeffrey Leonard Vickie Leonhardt Peter Lu Carol Maynard Kate McCracken Connor McDanel Emily McDowell Lauryn Michaels Peter Minnenci Dennis Minzler Larry Moss Erin Ney Kim Osborne Allison Pegg Gil Peri Jeb Phillips Ron Pontius Patrick Queen Raquel Quintanilla Amoros Carlos Reck Brenda Ruth Danielle Sabol

• • • • • • • • • • • • • • • • • • • •

Steve Sales Stephanie Savko Alicia Shoemaker Tori Smart Seryna Smith Rachael Subleski Benjamin Thompson Mandy Thompson Casey Thurston Bethanne Tilson Kathleen Tucker Karla Vaz Alejandra Vilanova Sanchez Jody Wall Renee Wells Carpenter Kara Wheeler Erin Willet Kent Williams Richard Wood Desalegn Yacob

xiii

PART

I

BOWEL MANAGEMENT

1

General guidelines for bowel management STEPHANIE J. VYROSTEK

Bowel management is an outpatient program used to treat fecal incontinence. The clinician selects an appropriate treatment modality, either oral stimulant laxatives or a mechanical program (once daily rectal or antegrade enema). The therapy is adjusted throughout the course of a devoted week according to the patient report and daily abdominal radiograph findings. Whether using an oral stimulant laxative or rectal enema washout, the goal of treatment in both cases is to stimulate a daily bowel movement and empty the colon. The child should then not have another bowel movement or soiling for 24 hours until the next treatment and thereby will be clean and able to wear normal underwear. The week begins with a contrast enema. This image provides information regarding the shape and size of the colon. Additionally, the speed at which a child evacuates contrast, as seen on subsequent abdominal radiographs, can provide valuable information from which one can infer the colonic motility. The contrast also cleans the colon of residual stool, giving the patient an empty colon to start the week of treatment. If after a contrast enema the patient still has a significant fecal burden, an at home bowel clean out is given prior to starting a mechanical program or laxative trial. The colon must be radiographically clean before starting either program.

 REATMENT MODALITY: T MECHANICAL PROGRAM OR STIMULANT LAXATIVES? The clinician should consider whether the patient has good or poor potential for bowel control when selecting 2

the treatment modality for the bowel management week. Patients with a good potential for bowel control typically respond very well to a stimulant laxative and water-soluble fiber therapies, but may benefit from rectal or antegrade enemas for a period of time depending on if there is dilation of the rectum and rectosigmoid that may impair rectal sensation or if they have never learned to have bowel control. They also become clean quickly and enjoy being clean, which sets them up for success when they try future laxative therapy. Patients with poor potential for bowel control benefit from a mechanical program (rectal or antegrade enema). Each regimen must be tailored to the needs of the patient, with particular attention paid to the age of the child, the underlying diagnosis (e.g., anorectal malformation, Hirschsprung disease, functional constipation, spinal conditions, sacrococcygeal teratoma), and the child’s “potential” for bowel control (i.e., their inherent ability to have voluntary bowel movements).

POTENTIAL FOR BOWEL CONTROL There are multiple factors that can be used to assess whether a patient has good potential for bowel control. These include maturity and a commitment to potty train, anal anatomy, anal sphincters, normal sacral anatomy, normal spinal anatomy (and thus normal innervation of the anorectum), and an anoplasty (surgically created anus) located in the center of the sphincter mechanism and without stricture or prolapse. We will review these six factors and how each can affect children with an anorectal malformation (ARM), Hirschsprung disease (HD), functional constipation, and spinal processes (see Figure 1.1).

Potential for bowel control

ARM

• • •

Type malformation Quality of sacrum Quality of spine

HD

• • •

Sphincters Dentate/anal canal Colonic motility

Spinal

• • •

Normal anal canal Normal sphincters Type of spinal lesion

Functional constipation

• • •

Normal anal canal Normal sphincters Normal spine

Figure 1.1 Predictors of continence. ARM  = anorectal malformation, HD = Hirschsprung disease.

MATURITY AND COMMITMENT TO POTTY TRAINING The age at which children begin potty training is different in a variety of countries and cultures. There are certain parts of the world that require the child to be potty trained to go to organized schooling such as preschool. This leads to increasing pressure on the parents to get their children in normal underwear as early as possible. Developmental delay also plays a significant role in whether a child is able to potty train when it is socially expected. Children with ARM, HD, functional constipation, and spinal processes will struggle with these societal “norms” as they try to potty train.

ANAL ANATOMY The anal canal and dentate line are vital for differentiating between solid stool, liquid stool, and gas. If there is damage to the anal canal, the child will be unsuccessful in recognizing the difference between the three and being able to pass them when socially acceptable. This could lead to stooling accidents. i. Children with an anorectal malformation almost

always lack a dentate line and normal anal canal as those failed to develop while in utero. These patients require a surgically created anal canal. ii. Those with Hirschsprung disease should have an intact dentate line when born; however, this can be damaged by the surgical dissection during a pullthrough surgery. iii. Children with functional constipation have a normal anal canal and dentate line, and those with spinal anomalies typically have a normal anal canal and dentate line, but, with the effect of poor nerve innervation, they may not function properly.

ANAL SPHINCTERS Intact sphincters are required for a child to have the ability to squeeze their anus; this includes both voluntary (skeletal) and involuntary (smooth) muscles. The anal sphincters may be relatively normal in children with an anorectal malformation who have a low malformation (perineal fistula, rectobulbar fistula, vestibular fistula, and short common channel cloaca), and slightly underdeveloped in those children with higher malformations (rectoprostatic fistula, bladder neck fistula, and long common channel cloaca). The quality of the sphincter in Hirschsprung disease is very good, but may have hypertonic smooth muscle that does not allow relaxation (absent RAIR—rectoanal inhibitory reflex) and permit stool to pass easily. Anal sphincters that are loose or overstretched in this patient population are typically of an iatrogenic cause. Children with functional constipation have normal anal sphincters, but can have overactive smooth muscles as well, which do not allow them to relax fully while attempting a bowel movement. In children with spinal anomalies, as with the anal canal, the anatomy is normal but the nerve innervation has been interrupted by their spinal anomaly. Disruption of anal sphincters can also be seen in patients who have suffered from pelvic trauma.

SACRAL ANATOMY The development of the sacrum seems to correlate with the development of the muscles and nerves in the pelvic floor. The underdevelopment of the sacrum typically is only associated with those children with anorectal malformations. In ARM patients, the quality of the sacrum, and therefore the muscles and nerves, can be assessed by calculating the sacral ratio (see Figure 1.2). A ratio greater than 0.7 is considered normal. One below 0.4 is quite hypodeveloped.

SPINAL ANATOMY The most frequent spinal anomalies treated with a bowel management program are spina bifida, tethered cord, occult spinal dysraphism, myelomeningocele, and sacrococcygeal teratomas. Spinal anomalies can be seen across all patients with ARM, HD, and functional constipation.

ANOPLASTY A surgically created anus in patients with anorectal malformations has to be placed within the center of the 3

General guidelines for bowel management

A

B C BC ------- = 0.77 AB

toilet to evacuate stool for 30 minutes. The type of stimulant, volume, and concentration are adjusted daily during the week as needed according to the patient’s tolerance, report of symptoms or soiling, and abdominal radiograph findings. The goal of treatment is to keep the child clean between enemas. See Table 1.1 for standard dosing and titration of saline, glycerin, Castile soap, and bisacodyl. Table 1.1  Enema formulation

GR

Saline base (rectal or antegrade) • Volume determined by provider based on contrast study • 20–30 mL/kg

GR

Glycerin enema (rectal or antegrade) • Use with saline base • Glycerin ordered in 5–10 mL increments • Once 40 mL has been reached, consider adding additional stimulant • If patient is symptomatic using this (e.g. vomiting/severe abdominal cramping), consider changing stimulant or substitute with Castile soap X-ray shows: (a) No stool burden: • Accidents: Decrease glycerin 10 mL

Normal ratio:

BC = 0.74 AB

• No accidents: Keep glycerin the same and continue to monitor

(b) Mild stool burden: Figure 1.2  Calculating a sacral ratio.

sphincter complex. This gives the child the best potential for continence as they grow. Even placement slightly out of the center can cause stooling accidents in patients who have the best potential. If there is a stricture or prolapse, this can interfere with successful continence and may need repair prior to potty training.

PROGRAM DETAILS: ENEMAS AND LAXATIVES ENEMAS A normal saline enema regimen (usually 15–20 mL/kg) is prescribed to the patient. Glycerin or a gentle soap are firstline stimulants to be added to the saline. Some children may require stronger stimulants such as bisacodyl, but should be maxed out on glycerin and soap prior to trying that medication. Children or their caregivers are instructed to administer the enema slowly over 5–10 minutes, hold the solution in for 5–10 minutes (if given rectally), and sit on the 4

• Accidents: Increase glycerin 10 mL • No accidents: Keep glycerin the same and continue to monitor

(c) Moderate stool burden: • Accidents/no accidents: Increase glycerin 10 mL

(d) Severe stool burden: • Accidents/no accidents: Increase glycerin 10 mL Castile soap enema (rectal or antegrade) use with saline base • Castile soap ordered in 9 mL increments • Once 36 mL has been reached, consider adding additional stimulant • If patient is symptomatic using this (e.g. vomiting/severe abdominal cramping), consider changing stimulant or substitute with baby soap X-ray shows:

(a) No stool burden: • Accidents: Decrease Castile soap 9 mL • No accidents: Keep Castile soap the same and continue to monitor

(b) Mild stool burden: • Accidents: Increase Castile soap 9 mL • No accidents: Keep Castile soap the same and continue to monitor (Continued)

Program details: Enemas and laxatives

Table 1.1 (Continued)  Enema formulation (c) Moderate stool burden: • Accidents/no accidents: Increase Castile soap 9 mL (d) Severe stool burden: • Accidents/no accidents: Increase Castile soap 9 mL Bisacodyl enema (rectal or antegrade) • Bisacodyl 10 mg/30 mL • Ordered in increments of 2.5 mg (7.5 mL) • Once 30 mg (90 mL) has been reached, consider adding additional stimulant • If patient is symptomatic using this (e.g. vomiting/severe abdominal cramping) consider changing stimulant X-ray shows: (a) No stool burden: • Accidents: Decrease bisacodyl 2.5 mg (7.5 mL) • No accidents: Keep bisacodyl the same and continue to monitor (b) Mild stool burden: • Accidents: Increase bisacodyl 5 mg (15 mL) • No accidents: Keep bisacodyl the same and continue to monitor (c) Moderate stool burden: • Accidents/no accidents: Increase bisacodyl 5 mg (15 mL) (d) Severe stool burden: • Accidents/no accidents: Increase bisacodyl 5 mg (15 mL)

LAXATIVES Senna-based oral stimulant laxatives are the first choice for treatment based on their ability to provoke the colon to contract and empty the stool burden. Stool softeners or osmotic laxatives (e.g.  polyethylene glycol) should be avoided because they make the stool soft, but do not provoke stool to empty. The laxative dose is determined from interpretation of the contrast enema, age and weight of the patient, and previously tried regimens. The patient is observed for 24 hours after laxative administration. If the child does not stool within 24 hours, their caregivers are instructed to administer an over the counter rectal enema to evacuate rectal stool burden. After the enema, the laxative dose is increased. In contrast, if the patient stools multiple times and the abdominal radiograph does not have a significant stool burden, the laxative dose can be decreased. Water-soluble fiber is prescribed in conjunction with laxatives to add bulk to the stool and to decrease the occurrence of watery stools. The laxative and fiber dosages are adjusted daily during the week as needed according to the patient’s tolerance, report of symptoms or soiling,

Table 1.2  Laxative protocol and fiber supplements Senna laxative protocol — after APN or MD assesses and classifies stool burden on X-ray • Senna ordered in 7.5 mg, 15 mg, or 25 mg increments • Max dose dependent on patient’s tolerance • Used in conjunction with water-soluble fiber X-ray shows: (a) No stool burden: • Accidents: Decrease laxative by half a tab or square (7.5 mg or 12.5 mg depending on dose) • No accidents: Keep laxative the same and continue to monitor (b) Mild stool burden: • Accidents: Keep laxative the same and consider increasing water-soluble fiber • No accidents: Keep laxative the same and continue to monitor (c) Moderate stool burden: • Accidents/no accidents: Increase laxative by a tab or square (15 mg or 25 mg depending on dose) (d) Severe stool burden: • Accidents/no accidents: Give rectal sodium phosphate enema (OTC, pediatric for children 2yo) AND increase laxative by tab or square (15 mg or 25 mg depending on dose) Water-soluble fiber protocol—after APN or MD assesses and classifies stool burden on X-ray • Water-soluble fiber ordered in 2 g increments • Max dose dependent on patient’s tolerance • Used alone or in conjunction with senna or loperamide X-ray shows: (a) No stool burden: • Loose stool and/or accidents: Increase water-soluble fiber 2 g • No accidents: Keep water-soluble fiber the same and continue to monitor (b) Mild stool burden: • Loose stool and/or accidents: Increase water-soluble fiber 2 g • No accidents: Keep water-soluble fiber the same and continue to monitor (c) Moderate stool burden: • Loose stool and/or accidents: Increase water-soluble fiber 2 g • No accidents: Keep water-soluble fiber the same and continue to monitor (d) Severe stool burden: • Accidents/no accidents: Give rectal sodium phosphate enema (OTC, pediatric for children 2yo) AND decrease water-soluble fiber by 2 g

and abdominal radiograph findings. The goal of treatment is one to two formed bowel movements per day and no soiling accidents. See Table 1.2 for standard dosing and titration of senna and water-soluble fiber supplementation. 5

Bowel management program setup: The basics and long-term follow-up

2

STEPHANIE J. VYROSTEK AND LAURA J. WEAVER

The creation and maintenance of a successful bowel management program (BMP) is an ongoing and evolving endeavor. Such a program consistently presents new challenges to overcome, which allows for institutional growth and development of the program. Some key drivers to identify prior to signing up patients should include: • Time frame (e.g., one-week program, monthly) • Number of patients and anticipated family participation • Institutional accommodations • Long-term follow-up We will discuss each point and provide recommendations for a successful program.

TIME FRAME Bowel management programs should include direct communication with patients and families for a minimum of 1 week. This time frame allows the provider caring for that patient to review applicable studies at the first meeting and troubleshoot any circumstances during the week to help the child achieve social continence. The provider may choose to see the child in clinic multiple times or to have email or phone communication throughout this 7-day period. A typical week may look like this (Figure 2.1): Day 1 • Contrast enema • Clinic visit

Day 2 • X-ray • Patient report

Figure 2.1  Typical week of bowel management.

6

Day 3 • X-ray • Patient report

NUMBER OF PATIENTS The amount of nursing support is vital, as nurses are the ones triaging all family inquiries. It is imperative not to schedule too many patients, as spreading the nurse too thin can cause less individual attention to the bowel management participants and cause them to feel unsupported, which could in turn negatively impact their outcomes. Having this direct contact and support network makes the patient and family feel valued. The number of patients a program can handle will need to be discussed with nursing staff and administrators, and constantly re-evaluated to make sure goals are being met in order to help as many children as possible, without overwhelming the staff or infrastructure. Family participation is vital to the success of the child’s bowel management program. The entire family has to be on board and needs to understand the “burden” of this therapy required to achieve success. Families need to be set up for success with realistic expectations for their child’s potential for continence. The family reports of how things are going when the child is not in clinic are vital for the providers who use this information, as well as the radiograph images to adjust medications. The data provided must be detailed and allow your team to know exactly what has happened at home during the Day 4 • X-ray • Clinic visit

Day 5 • X-ray • Patient report

Day 6 • X-ray • Patient report

Day 7 • X-ray • Clinic visit

Institutional accommodations

Daily enema program chart 1st Thursday

Friday

Normal saline amount (mL) Glycerin amount (mL) Castile soap amount (mL) Other medicines (name and dose) Diet instructions

Time enema started Time enema finished How long enema took to go in (minutes) How long enema stayed in (minutes) Any symptoms when giving enema? Total time seated on the toilet (minutes) Accidents: How many? Time? Description?

Figure 2.2  Enema program report sheet.

individualized program. See Figures 2.2 through 2.4 for examples of family report sheets for the enema program and the laxative programs.

INSTITUTIONAL ACCOMMODATIONS A bowel management program will likely bring an increase of patients to a clinic, and this will also affect the institution as a whole. These patients will impact

radiology (ultrasound, magnetic resonance imaging [MRI], interventional radiology), pharmacy, urology, gynecology, gastroenterology and neurosurgery, as well as the operating room. Including each team in the discussion of flow and impact is essential to success and each department has their own part to play in the success of the program. Having the support of the institution and all the other departments will help the patients feel valued across the organization. Preliminary meetings with all involved parties is a key step in the process. 7

Bowel management program setup: The basics and long-term follow-up

Daily medicine program for hypermotility chart 1st Thursday

Friday

Kind and amount of medicine Time medicine given Kind and amount of fiber Diet instructions Voluntary bowel movements: Amount and description Accidents/soiling: Time, amount and description

Figure 2.3  Medicine program for hypermotility.

Daily medicine program for hypomotility chart 1st Thursday Kind and amount of laxative Time laxative given Kind and amount of fiber Other medicines (name and dose) Diet instructions Voluntary bowel movements: Amount and description Accidents/soiling: Time, amount and description

Figure 2.4  Medicine program for hypomotility.

8

Friday

Long-term follow-up

LONG-TERM FOLLOW-UP When the week-long program has finished, there is still ongoing management that must be maintained. Losing sight of the long-term efforts that need to go into this patient population is a recipe for poor outcomes. Initially, close follow-up is required to confirm that the patient’s regimen is continuing to work and all concerns the family may be facing upon returning home can be addressed. This is routinely done at 1 month and 3 months, after the completion of the bowel management program. The goal would be then to move to annual follow-up until any future intervention is needed. In the authors’ center, the provider completes the follow-up intervals at 1 month and 3  months in clinic or over the phone and the annual follow-up in person in clinic. Prior to each follow-up appointment, the patient gets an abdominal X-ray. They also complete a questionnaire about their regimen and certain pertinent

Urinary questions

continent and quality of life scores. The scores currently used are the Baylor Social Continence Scale, Cleveland Clinic Constipation Scoring System, and Vancouver Dysfunctional Elimination Syndrome Survey with the addition of Pediatric Quality of Life at the annual visits (Figure 2.5). The provider reviews the X-ray image and the outcomes of the patient’s scores and then conducts the clinic visit or the phone interview with the family. If at any time the provider feels the need to adjust the follow-up schedule, it is done while still collecting the completed scores at the predetermined intervals (Figure 2.6). The Baylor Social Continence Scale, Cleveland Clinic Constipation Scoring System, Vancouver Dysfunctional Elimination Syndrome Survey, and Pediatric Quality of Life are also collected prior to the patient completing the bowel management program. This provides the opportunity to see the patient’s objective progress, as well as areas for growth. See Figure 2.7 for an example of a bowel management flowsheet for follow-up care.

Asked by the provider

• Any changes to your child’s urinary status? • Any UTI or UTI symptoms? Stooling questions

Asked by the provider and assessed on abdominal X-ray

• How many bowel movements per day? • Are they accidents or voluntary? • What time of day do they occur? Regimen questions

Assessed and changed by the provider

• What is the current regimen your child is on? • Is your child tolerating that regimen? • Are there any barriers you have experienced since being home? Figure 2.5  Example question for follow up.

1 month (virtual option)

3 month (virtual option)

• Baylor

• Baylor

• Baylor

• Baylor

• Cleveland

• Cleveland

• Cleveland

• Cleveland

• Vancouver

• Vancouver

• Vancouver

• Vancouver

Intake (virtual option)

• Peds QL

1 year

• Peds QL

Figure 2.6  Survey collection intervals.

9

Bowel management program setup: The basics and long-term follow-up

SUGGESTED READINGS

Bowel management program (BMP)

First intervention

2nd Thursday schedule out BMP track appointment

Previous BMP

Previous surgery

2nd Thursday

2nd Thursday Stop surgical track and schedule out new BMP track

Stop previous BMP track and schedule out new BMP track

1 month

1 month

1 month

Outcomes

Outcomes

Outcomes

3 month

3 month

3 month

Outcomes

Outcomes

Outcomes

12 month

12 month

12 month

Outcomes

Outcomes

Outcomes

2–5 year

2–5 year

2–5 year

Outcomes

Outcomes

Outcomes

Figure 2.7  Bowel management flowsheet for follow-up care.

10

Afshar, K., Mirbagheri, A., Scott, H., & MacNeily, A. E. 2009 October. Development of a symptom score for dysfunctional elimination syndrome. Journal of Urology, 182(4 Suppl), 1939–1943. Agachan, F., Chen, T., Pfeifer, J., Reissman, P., & Wexner, S. D. 1996 June. A constipation scoring system to simplify evaluation and management of constipated patients. Diseases of the Colon Rectum, 39(6), 681–685. Bischoff, A., & Levitt, M. 2009. Treatment of fecal incontinence with a comprehensive bowel management program. Journal of Pediatric Surgery, 44, 1278–1284. Brandt, M. L., Daigneau, C., Graviss, E. A., Naik-Mathuria, B., Fitch, M. E., & Washburn, K. K. 2007 June. Validation of the Baylor Continence Scale in children with anorectal malformations. Journal of Pediatric Surgery, 42(6), 1015–1021; discussion 1021. Levitt, M. A., & Pena, A. 2010. Pediatric fecal incontinence: A surgeon’s perspective. Pediatrics in Review, 31, 91–98. Reck-Burneo, C. A., Vilanova-Sanchez, A., Gasior, A. et al. 2018. A structured bowel management program for patients with severe functional constipation can help decrease emergency department visits, hospital admissions, and healthcare costs. Journal of Pediatric Surgery, 53(9), 1737–1741. Vilanova-Sanchez, A., Halleran, D. R., Reck-Burneo, C. A. et al. 2019 March. A descriptive model for a multidisciplinary unit for colorectal and pelvic malformation. Journal Pediatric Surgery, 54(3), 479–485.

ANORECTAL MALFORMATIONS

PART

II INTRODUCTION Onnalisa Nash

Anorectal malformation (ARM) describes the congenital problem of an absent anal opening or one that is in the wrong place. These anomalies are a spectrum of disorders involving the distal anus, rectum, and frequently the urinary and gynecologic system. The prevalence is about 1 in 5000 births worldwide. After surgical repair, patients frequently require treatment for constipation and/or fecal incontinence. Fecal and urinary incontinence can occur even with an excellent anatomic repair usually related to associated spinal or pelvic hypo or abnormal development. An effective bowel management program is the most beneficial way to improve the quality of life of a patient with these concerns. The most frequent morbidity encountered after surgical repair of an ARM is constipation. It is also the most important problem to avoid after definitive repair, starting right after repair or after colostomy closure as the case may be. If constipation is not avoided and well treated postoperatively, an enlarged rectum and sigmoid can result, which can lead to fecal impaction and overflow incontinence. Less frequent than constipation, some patients experience soiling. In a patient with a good prognosis, this may manifest as overflow incontinence (meaning the patient has the potential for bowel control but soils due to regular impactions). It may also represent true fecal incontinence (meaning the patient lacks the ability to have a voluntary bowel movement) in cases of very high malformations, and when there is an associated spinal abnormality and/or an abnormal sacrum. A contrast enema along with the sacral ratio and spinal imaging is helpful in discerning this information. The ARM Index (Figure II.1) is helpful in predicting continence in patients with ARM.

Anorectal malformations

ARM INDEX

SACRUM

SPINE

ARM TYPE

POINTS Perineal Fistula

1

Rectal Stenosis

1

Rectal Atresia

1

Rectovestibular Fistula

1

Rectobulbar Fistula

1

lmperforate Anus Without Fistula

1

Cloaca < 3 cm Common Channel

2

Rectoprostatic Fistula

2

Rectovaginal Fistula

2

Recto–bladder neck Fistula

3

Cloaca > 3 cm Common Channel

3

Cloacal Exstrophy

3

Normal Termination of the Conus (L1–L2)

1

Normal Filum Appearance

1

Abnormally Low Termination of the Conus (Below L3)

2

Abnormal Fatty Thickening of Filum

2

Myelomeningocele

3

Sacral Ratio = Greater than 0.7

1

Sacral Ratio = Between 0.4 and 0.69

2

Hemisacrum

2

Sacral Hemivertebrae Presacral Mass

2 2

Sacral Ratio = Less than 0.4

3

TOTAL POINTS

3–4 = Good Potential for continence 5–6 = Fair Potential for continence 7–9 = Poor Potential for continence

Figure II.1  Arm Index.

The ARM Index shown has a “report card” format that can help predict continence potential in a patient with ARM. The three areas that are assessed are the type of malformation, the quality of the spine, and the quality of the sacrum. Using the ARM Index report card, points are given in each area, which are then added to determine the score for each patient. The patients with good potential for continence will have a score of 3–4, fair potential for continence will have a score of 5–6, and poor potential will have a score of 7–9. Once the ARM index score is determined, the prognosis for continence can be predicted. If a patient’s ARM Index indicates good potential for continence, the practitioner and family can plan for the probability of success with a laxative regimen. If a patient’s diagnosis points to a poor prognosis, the practitioner and parents should plan for the likelihood that that child will need a mechanical bowel regimen to remain clean. Patients with fair potential usually need to be started on a mechanical regimen at first, but may be able to transition to a medical regimen (laxatives) as they grow older. As the child matures, the likelihood of achieving bowel control generally improves. 12

The cases we will consider in this section include: • A patient with good surgical anatomy with good potential for bowel control • A patient with good surgical anatomy with fair potential for bowel control • A patient with good surgical anatomy with poor potential for bowel control • A patient in need of bowel management with the need for urologic reconstruction • A young adult with prior surgery for ARM, with fecal incontinence • A patient with ARM who is a sacral nerve stimulator candidate

SUGGESTED READINGS Lane, V., Wood, R., Reck, C., & Levitt, M. 2017. Pediatric Colorectal and Pelvic Surgery: Case Studies. CRC Press. Wood, R., & Levitt, M. 2018. Anorectal malformations. Clinics in Colon and Rectal Surgery. 31(02), 061–070.

3

A patient with good surgical anatomy after an anorectal malformation (ARM) repair with good potential for bowel control ERIN M. SHANN

CASE HISTORY A 6-year-old boy, adopted from China at the age of 3, comes to your clinic. He was born with an anorectal malformation (perineal fistula) that was repaired as a newborn in China as a primary posterior sagittal anorectoplasty (PSARP). He now has daily soiling with his home enema regimen of 400 mL normal saline, 15 mL glycerin, and 10 mL Castile soap. His spine and urological system are normal and his lateral sacral ratio is 0.78 (refer to the figure “ARM Index” in the color insert).

A contrast enema was performed and is shown in Figures 3.1 and 3.2. A post-evacuation film is shown in Figure 3.3.

QUESTION 3.1 What is your assessment of his potential for continence?

Figure 3.1  Contrast enema, lateral view.

A. Good B. Fair C. Poor Answer: This patient had a perineal fistula repaired, which is a low malformation. He has no abnormalities with his sacrum and no signs of tethered cord. His sacral ratio is within the normal range, which is greater than 0.7. Based on this information, the patient has a good potential for bowel control (Answer = A). During the initial evaluation of the patient, physical exam findings were normal (the anoplasty was well placed, without stricture or rectal prolapse), and a contrast study was performed to assess the anatomy. The prior surgery was done well.

Figure 3.2  Contrast enema, frontal view.

13

A patient with good surgical anatomy after an ARM repair with good potential for bowel control

QUESTION 3.4 What should the next steps be? A. Stay on rectal enemas B. Get urology clearance and perform a Malone appendicostomy C. Start laxatives and fiber D. Attend a bowel management week

Figure 3.3  Post-evacuation radiograph.

QUESTION 3.2 What does the contrast study show? Answer:  The contrast study shows a mildly redundant sigmoid colon and dilated rectum.

Answer: A Malone appendicostomy was performed (Answer = B) given his age, his persistent soiling, and intolerance of rectal enemas due to him being anally defensive. Despite him having good potential for bowel control, we still felt a mechanical program was best at this point in his life. Once this is achieved, it could be the bridge to future continence using laxatives. Before he was able to have the Malone appendicostomy performed, he needed to receive the “golden ticket” from urology. The “golden ticket” is when urology determines the appendix is not needed for a Mitrofanoff or any other urological repair. If that were to be the case the urologic procedure and the Malone can be done at the same operation. All of his urology testing was normal and he needed no urological surgery done at the same time as the Malone. At one month postoperatively from the Malone, he attended a bowel management week.

QUESTION 3.3 What regimen would you start? A. Laxatives and fiber B. Rectal enemas Answer:  Based on his first evaluation, he was kept on rectal enemas (Answer = B) due to consistently having soiling with his current home regimen of 400 mL normal saline, 15 mL glycerin, and 10 mL Castile soap. He was kept on rectal enemas and not switched to a laxative and fiber regimen because he has never been “clean” of stool and his contrast study showed dilation of the rectum. We changed him to a regimen of 350 mL normal saline, 20 mL glycerin, and 15 mL Castile soap to see if this would help the colon become clean daily and allow him to avoid soiling. The strength of his flush was increased to see if it would provoke a larger bowel movement at enema time. Despite this change, stool accidents occurred approximately 18 hours after the rectal enema was administered and the patient had begun to become resistant to the enema administration.

14

DAY 1 Patient report: • Regimen: 350 mL normal saline, 20 mL glycerin , and 15 mL Castile soap • An abdominal X-ray was performed (Figure 3.4)

Figure 3.4  Day 1 X-ray.

Case history

QUESTION 3.5

QUESTION 3.7

What does the X-ray show?

What does the X-ray show?

A. The X-ray is clean B. Small amount of stool throughout the colon C. Moderate stool in the ascending colon and the rectum D. Stool only in the rectum

A. Stool only in the ascending colon B. Clean X-ray, no stool C. Stool in the ascending colon but clean throughout

Answer:  There is moderate stool in the ascending colon and in the rectum (Answer = C). The colon needs to be cleaner.

Answer: The X-ray is very clean now which means the flush is strong. There is a small amount of stool in the ascending colon. There is no stool accumulation in the transverse, descending colon or rectum (Answer = C).

QUESTION 3.6 What would be your new recommended regimen? A. 400 mL normal saline and 20 mL glycerin B. 400 mL normal saline, 20 mL glycerin, and 9 mL Castile soap C. 350 mL normal saline, 20 mL glycerin, and 15 mL Castile soap D. 450 mL normal saline, 20 mL glycerin, and 15 mL Castile soap Answer:  The patient was placed on 450 mL normal saline, 20 mL glycerin, and 15 mL Castile soap (Answer = D). Normal saline was increased to help fill the colon more, thereby helping the colon to empty.

QUESTION 3.8 Would you change the regimen? A. The flush is just right, no changes B. Too strong of a flush, decrease the strength of the regimen C. The flush is not strong enough, increase the strength of the regimen Answer: It is recommended to decrease the strength of the flush slightly to help avoid accidents that are being provoked by the stronger flush (Answer = B).

DAY 2

DAY 3

Patient report:

Patient report:

• Regimen: 450 mL normal saline, 20 mL glycerin, and 15 mL Castile soap • One accident at 19 hours after the flush with some leaking (Figure 3.5)

• Regimen: 400 mL normal saline, 20 mL glycerin, and 9 mL Castile soap • Family reported an overnight accident

QUESTION 3.9 A new X-ray was obtained. What does it show (Figure 3.6)? Answer:  There is a small amount of stool in the ascending colon which is of no significance. There is still no stool in the transverse, descending colon, or the rectum. This X-ray is again very clean.

Figure 3.5  Day 2 X-ray.

15

A patient with good surgical anatomy after an ARM repair with good potential for bowel control

Figure 3.6  Day 3 X-ray. Figure 3.7  Day 4 X-ray, view 1.

QUESTION 3.10 What regimen ingredient would you change? A. Normal saline B. Glycerin C. Castile Answer: The patient is being overstimulated with the additives glycerin and Castile soap. The patient was taken off of Castile soap because it has a higher stimulation effect on the colon (Answer = C).

DAY 4

Figure 3.8  Day 4 X-ray, view 2.

Patient report: • Regimen: 400 mL normal saline and 20 mL glycerin • Family reported a small overnight accident along with a small daytime accident. • X-ray shows: Stool in the descending colon and in the rectum (Figures 3.7 and 3.8).

16

QUESTION 3.11 What does the X-ray show? Answer:  The X-ray now shows stool in the ascending and descending colon, as well as the rectum.

Case history

QUESTION 3.12

QUESTION 3.14

What would you like to do next?

How would you change the regimen?

A. Increase normal saline by 50 mL B. Decrease glycerin by 10 mL C. Add fiber to the current regimen D. Take glycerin out of the regimen

A. Keep the regimen the same to give the fiber a chance to help bulk the stool more B. Take the fiber away, it bulked the stool too much C. Add 9 mL of Castile soap into the regimen D. Increase the glycerin by 10 mL

Answer: The patient still needs the additive of the glycerin to help stimulate the colon to provoke a bowel movement. The X-rays have been cleaner, but the patient is still experiencing fecal soiling. Fiber was added to the regimen to help bulk the stool along with keeping the flush of 400 mL normal saline and 20 mL glycerin (Answer = C).

DAY 5 Patient report: • Regimen: 400 mL normal saline, and 20 mL glycerin, and 2 g of fiber twice a day • Family reported an overnight accident and another accident about 22–24 hours after previous flush (Figure 3.9)

Answer:  Glycerin was increased by 10 mL (Answer = D) because the patient was accumulating stool in the descending colon. When a patient is on antegrade flushes or rectal enemas, the goal is to have the descending colon clean of stool.

DAY 6 Patient report: • Regimen: 450 mL normal saline, 30 mL glycerin, and 2 g fiber twice a day • Continues to experience 1–2 episodes of leakage overnight (Figure 3.10)

Figure 3.9  Day 5 X-ray.

Figure 3.10  Day 6 X-ray.

QUESTION 3.13

QUESTION 3.15

What does the X-ray show?

What does the X-ray show?

Answer: The X-ray shows stool accumulation in the ascending and descending colon.

Answer:  The X-ray demonstrates stool accumulation in the rectum.

17

A patient with good surgical anatomy after an ARM repair with good potential for bowel control

QUESTION 3.16

KEY LEARNING POINTS

It is the last day of bowel management. What will you change the regimen to?

1. Malone appendicostomies can be considered for those who are anally defensive to help decrease the psychological component with doing rectal enemas for both the patient and the family. 2. Soiling after the flush with a clean X-ray could mean that the flush is too stimulating and is provoking accidents between flushes. 3. Fiber can be added to the regimen to help bulk the stool to decrease the number of accidents the patient is experiencing. 4. A patient with good potential for bowel control is likely to succeed on a laxative and fiber regimen. 5. If the patient is a good candidate for a laxative and fiber trial, the ultimate decision on when to do this will depend on the family and patient’s readiness.

A. Continue same regimen and give the fiber time to bulk the stool B. Increase the fiber to three times a day C. Decrease the normal saline, decrease the glycerin, add Castile soap, and take away the fiber Answer:  The regimen for home is 400 mL normal saline, 20 mL glycerin, and 5 mL Castile soap (Answer = C). The fiber was taken away due to stool accumulation on the X-ray. The glycerin was decreased and the Castile soap was added back into the regimen to help provoke more of a bowel movement with the combination of the stimulants.

PRESENT DAY During a phone follow-up appointment, the family reported that the patient was currently doing 400 mL normal saline, 20 mL glycerin, and 9 mL Castile soap via Malone. The patient rarely had accidents, approximately three times per month. The family reported that they occasionally skip a flush over the weekend.

QUESTION 3.17 Based on how bowel management went and the present day, what would the next steps be for this patient based on their anatomy and their potential for bowel control? A. Continue same regimen B. Have patient begin every other day flushes C. Have patient attend another bowel management week to transition to laxatives Answer:  The family decided to stay on the same daily flush regimen at this time. The family likes the convenience of only having one bowel movement per day. They will consider attending a formal bowel management week for a laxative and fiber trial. Given the patient’s good potential for bowel control they would do well on a laxative and fiber trial (Answer = C).

18

SUGGESTED READING Reck-Burneo, C. A., Vilanova-Sanchez, A., Gasior, A. C. et al. 2018. A structured bowel management program for patients with severe functional constipation can help decrease emergency department visits, hospital admissions, and health care costs. Journal of Pediatric Surgery, 53, 1737–1741.

4

A patient with good surgical anatomy following an anorectal malformation (ARM) repair with fair potential for bowel control ERIN M. SHANN

CASE HISTORY

QUESTION 4.1

A 4-year-old boy comes to your clinic with a history of anorectal malformation (ARM) (rectoprostatic urethral fistula), tethered cord, and sacral dysgenesis, complaining of daily accidents. His initial ARM repair was done at an outside hospital in his first year of life with preceding colostomy as a newborn and subsequent colostomy closure. His current bowel regimen includes lactulose, polyethylene glycol 3350 (MiraLAX), and Fleet enemas to treat constipation.

What is your assessment of his potential for continence? A. Need more testing to help determine true potential for continence B. Good potential for continence Answer: More testing is needed to determine the true potential for continence (Answer = A).

QUESTION 4.2 What testing would you need? Answer: A sacral X-ray and a magnetic resonance imaging (MRI) spine are needed to assess his continence potential (Figure 4.1). Predictors of continence in ARM

Type of malformation

Quality of sacrum

Status of spine

Figure 4.1  Predictors of continence in ARM needed to assess such a patient.

19

A patient with good surgical anatomy following an ARM repair with fair potential for bowel control

QUESTION 4.3 What are the next steps? A. Continue current regimen with no additional workup B. Complete a formal bowel management week with a contrast study C. Complete a formal bowel management week with a contrast study, renal testing, and an MRI spine D. Continue current regimen, obtain a contrast study, renal testing, and an MRI spine Answer: The patient should complete a formal bowel management week, obtain a water soluble contrast study, a renal ultrasound, and a pelvic MRI (Answer = C). A water soluble contrast study is needed to assess the anatomy of the colon to check for redundancy, distention, twists, narrowing, and a widened pre-sacral space (which could indicate a pre-sacral mass). The renal ultrasound is needed to check the health of the kidneys. It is found that most individuals with ARM are prone to have poor kidney health. The pelvic MRI is needed to check for a pre-sacral mass and a remnant of the original fistula (ROOF).

Figure 4.3 Contrast enema, frontal view, showing the rectosigmoid.

His contrast enema is shown in Figures 4.2 through 4.4.

Figure 4.4  Contrast enema, frontal view, showing entire colon.

The post-evacuation film is shown in Figure 4.5.

Figure 4.2  Contrast enema, lateral view.

Figure 4.5  Post-evacuation film.

20

Bowel management week

QUESTION 4.4

QUESTION 4.7

What does the contrast study show?

What regimen would you start?

Answer: The contrast study shows a redundant rectosigmoid colon. The post-evacuation film shows partial emptying of colonic contrast with residual contrast remaining throughout the colon. Upon exam in the clinic, the patient had a slight posteriorly located anoplasty. His spinal MRI revealed him to have a fatty filum without tethering. His sacral ratio is AP: 0.41, Lat: 0.69. The renal ultrasound showed normal kidneys with a mild post-void residual, and a 2 cm cystic structure adjacent to the posterior right aspect of the urinary bladder.

A. Rectal enemas B. Continue current regimen of MiraLAX, lactulose, and Fleet enemas C. Laxatives and fiber D. Bisacodyl

QUESTION 4.5 What do you think this cystic structure could be? A. Possibly a remnant of the original fistula (ROOF) B. A bladder stone C. A ureterocele Answer: The pelvic MRI confirmed the suspicion that this cyst was in fact a very small remnant of the original distal rectum attached to the posterior urethra (Answer = A). This did not require any surgical intervention.

QUESTION 4.6 What would the patient’s potential for bowel continence be? A. Good B. Fair C. Poor Answer: The ARM Index is a tool utilized to help gauge an individual’s level of bowel continence. It is based on a number system which categorizes individuals into good, fair, or poor potential for continence (refer to the figure “ARM Index” in the color insert). This patient’s ARM type is a rectoprostatic fistula (2 points). His spine revealed abnormal fatty thickening of filum, which adds an additional 2 points. His sacral ratio is 0.69, which will bring his final number to 6. Based on utilizing the ARM Index, this patient’s potential for bowel continence is fair (Answer = B).

Answer: Based on his first evaluation, he was placed on rectal enemas (Answer = A). These were started because the colon shows distention and the child was having frequent stool accidents. This would indicate that he is not on the correct bowel regimen at present. The patient was not started on a laxative and fiber regimen because his colon needed time to shrink down to a manageable size. If the colon is too distended when you start a laxative and fiber regimen, the dose needed to provoke a proper bowel movement would be a higher dose. Starting with a higher dose could cause an increase in abdominal cramping, which could deter the patient from continuing the medication in the beginning.

QUESTION 4.8 Why are rectal enemas typically started first? Answer: If the patient has good or fair potential for bowel continence, rectal enemas are a way to reset the colon to allow for the colon distention to shrink to a normal sized colon. Giving the rectal enemas for 6 to 12 months will allow this to occur. Starting with rectal enemas is a “quick fix” to help promote social continence. The patient begins to learn what it is like to be clean and not have stool accidents. This will provide the patient with motivation if laxatives and fiber are tried in the future.

BOWEL MANAGEMENT WEEK DAY 0 Patient report: • Regimen: 1 cap MiraLAX twice a day, 2 tablespoons lactulose daily, a Fleet enema as needed • Family reports 1–2 accidents per day and 1 voluntary bowel movement every other day with the help of a Fleet enema (Figure 4.6) 21

A patient with good surgical anatomy following an ARM repair with fair potential for bowel control

QUESTION 4.9 What does the X-ray show? Answer: The X-ray shows stool distending the colon from the cecum to the rectum.

DAY 2 Patient report: • Regimen: 500 mL normal saline, 30 mL glycerin, and 9 Castile soap • No reported accidents (Figure 4.8)

Figure 4.6  Day 0 X-ray.

The recommended regimen to start bowel management was 500 mL normal saline and 30 mL glycerin based off the contrast volume amount and to stop the MiraLAX, lactulose, and Fleet enemas.

DAY 1 OF RECTAL ENEMAS Patient report: • • • •

Regimen: 500 mL normal saline and 30 mL glycerin Voluntary bowel movement before nightly enema One accident 2 hours after the enema No complaints of pain or cramping (Figure 4.7)

Figure 4.8  Day 2 X-ray.

QUESTION 4.10 What does the X-ray show? Answer: The X-ray shows an increase in gas, but a decrease in stool.

QUESTION 4.11 What would you change about the regimen?

Figure 4.7  Day 1 X-ray.

22

A. Continue the current regimen B. Stop glycerin and increase Castile soap C. Stop Castile soap D. Switch Castile soap to baby soap

Bowel management week

Answer: The regimen was changed to 500 mL normal saline and 27 mL Castile soap. The glycerin was stopped and the Castile soap was increased (Answer = B) because glycerin can cause an increase in gas in some patients.

DAY 3 Patient report: • Regimen: 500 mL normal saline and 27 mL Castile soap • Family reported one small overnight accident • Family reported a decrease in abdominal cramping (Figure 4.9)

Answer: The normal saline was decreased to help strengthen the concentration of the Castile soap since the patient had a small overnight accident (Answer = C). The new regimen is 450 mL normal saline and 27 mL Castile soap.

DAY 4 Patient report: • Regimen: 450 mL normal saline and 27 mL Castile soap • Small accident reported a half hour after rectal enema • Small overnight accident reported (Figure 4.10)

Figure 4.10  Day 4 X-ray. Figure 4.9  Day 3 X-ray.

QUESTION 4.14

QUESTION 4.12

What does the X-ray show?

What does the X-ray show?

Answer: The X-ray shows stool throughout the colon in the ascending, descending, and rectum.

Answer: Less gas is seen on this X-ray. Stool is in the ascending and transverse colon.

QUESTION 4.15 QUESTION 4.13 What would you change the regimen to? A. Continue the same regimen B. Change Castile soap to baby soap C. Decrease the normal saline D. Add glycerin back into the regimen

What do you want to do with the regimen? A. Continue the same regimen and review administration technique B. Decrease normal saline C. Change Castile soap to baby soap D. Decrease Castile soap and review administration technique

23

A patient with good surgical anatomy following an ARM repair with fair potential for bowel control

Answer: The administration technique was reviewed with the family and the Castile soap was decreased because the patient continued to have small accidents (Answer = D). The actual technique performed with rectal enemas is important. If one area during the administration process, that is, infusion rate of the solution, hold time of the solution, or not sitting on the toilet for the recommended 45 minutes, is performed differently than the recommendations of the health care provider, this can make the enema ineffective.

DAY 5 Patient report: • Regimen: 450 mL normal saline and 18 Castile soap • Small accident overnight (Figure 4.11)

QUESTION 4.17 What would you change the regimen to? A. Continue the same regimen B. Change the Castile to baby soap C. Decrease the normal saline Answer: The regimen was changed to 400 mL normal saline and 18 mL Castile soap. The normal saline was decreased (Answer = C) because the patient was still experiencing accidents. Decreasing the normal saline should help make the flush less diluted and the flush stronger. An exam under anesthesia with cystoscopy was performed during bowel management week to assess the anatomy and to look for a ROOF hinted at on the ultrasound and pelvic MRI. The urinary system was found to be normal and there were no signs of a ROOF. The anus was posteriorly mislocated (outside the sphincter mechanism) by 2 cm.

QUESTION 4.18 What are the next steps? A. Nothing, no surgical intervention is needed B. Surgical intervention is needed to relocate the anus and create a Malone appendicostomy C. Surgical intervention is needed to relocate the anus D. Surgical intervention is needed for a Malone appendicostomy and Mitrofanoff only

Figure 4.11  Day 5 X-ray.

QUESTION 4.16 What does the X-ray show? Answer: The X-ray shows stool throughout the colon in the ascending, descending, and rectum. There is no change in stool volume compared to the previous X-ray.

24

Answer: At the completion of bowel management week, the patient was scheduled for a reoperation (redo) via a posterior sagittal anorectoplasty (PSARP) approach to move the anus to the center of the sphincters and a laparoscopic Malone appendicostomy to allow for antegrade flushes (Answer = B). During bowel management week, the patient started to resist undergoing the rectal enemas. It was decided to proceed with the Malone appendicostomy because this patient would most likely need to be on either a rectal enema regimen or an antegrade regimen to help keep him be clean of stool accidents throughout the day for longer than a 6-month period. The patient received urology clearance to proceed with the Malone only. The patient did not need a Mitrofanoff, because his urological system was normal and he was experiencing no symptoms that would indicate this intervention. Therefore, there was no need to consider that the appendix would be needed one day to be shared for a Malone and Mitrofanoff.

Suggested reading

QUESTION 4.19 Why was a regimen started before the redo? Answer: It is important to have a good regimen for the patient prior to the redo PSARP and Malone appendicostomy to allow the patient to begin to have social continence quickly and to allow the colon to begin to decrease in size. A PSARP with a Malone appendicostomy was performed a few months after the completion of bowel management week. At a 1-year follow-up with the patient, the regimen was 500 mL normal salin, 30 mL glycerin, and 27 mL Castile soap. The family reported no accidents with this regimen and the patient was tolerating the antegrade flushes well. A few years later after being on antegrade flushes, the family asked to transition to laxatives. When the patient is older they can “listen” to their body as to when it is time for a bowel movement.

QUESTION 4.20 What would your next steps be?

The plan for the patient is to continue their current laxative dosage and increase the fiber dosage to 9 g twice a day. The family is to do an antegrade flush once a week in the hope of “resetting” the colon to help decrease the number of accidents.

QUESTION 4.21 When do you consider switching back to antegrade flushes? Answer: This decision is up to the family on when or if they would like to switch back to antegrade flushes, in this case possibly a more reliable way to keep the patient clean.

QUESTION 4.22 When do you discuss with the family the patient’s quality of life with stool accidents? Answer: This is a continued discussion with the family through clinic appointments, emails, and via phone calls. Psychology should be consulted based on provider discretion.

A. The patient needs more time on antegrade flushes B. Attend a laxative bowel management week Answer: The patient attended a formal laxative trial during a bowel management week (Answer = B). At the beginning of the week, the patient’s regimen consisted of 75 mg of ex-lax once a day with 3 g of Nutrisource fiber twice a day. At the completion of this bowel management week, the regimen consisted of 125 mg of ex-lax daily with 9 g of Nutrisource in the morning and 6 g of Nutrisource in the evening.

PRESENT DAY During a follow-up visit, the family reported that the patient was taking 137.5 mg of senna tablets per day and 7.5 g of Nutrisource fiber twice a day. The patient was having four to six small voluntary bowel movements per day. The family reported that every one to two weeks, the patient will go 24 hours without a bowel movement and then will proceed to have several accidents for a few days. The family also reported that they will do an antegrade flush if there has been no stool output within 24 hours.

KEY LEARNING POINTS 1. If the colon shows distention on the contrast study, rectal enemas are started to help the colon shrink down to a normal size. This is done over a 6- to 12-month period. 2. Normal saline can be decreased to help strengthen the regimen by concentrating the additives. 3. Administration technique is important to review with families at the beginning of bowel management week and when they go home in order to set them up for success.

SUGGESTED READING Reck-Burneo, C. A., Vilanova-Sanchez, A., Gasior, A. C. et al. 2018. A structured bowel management program for patients with severe functional constipation can help decrease emergency department visits, hospital admissions, and health care costs. Journal of Pediatric Surgery, 53, 1737–1741. 25

5

A patient with a good anatomic anorectal malformation (ARM) repair, but with poor potential for bowel control CATHERINE TRIMBLE

CASE HISTORY A 5-year-old male presents to your clinic with a history of an anorectal malformation (ARM). As a newborn, a colostomy was opened and he had a rectouretheral (bladder neck) fistula for which a laparoscopic-assisted posterior sagittal anorectoplasty (PSARP) was performed. Thereafter, the colostomy was closed. The parents were told the child would be fine and no routine follow-up or any other treatments were necessary. At age 5, the child presents to you with daily soiling.

QUESTION 5.1 What do you predict is the cause of the soiling?

QUESTION 5.2 What evaluation do you suggest? Answer: Spinal MRI, sacral X-rays, exam under anesthesia, contrast enema.

Spinal MRI reveals termination of the conus at L4, consistent with a tethered cord. Neurosurgery evaluated him for this and performed a cord detethering. Sacral ratio AP: 0.58; lateral: 0.66. Examination revealed an anus well centered within the sphincter complex, with no stricture or prolapse. A contrast enema via the rectum was obtained (Figure 5.1)

Answer: The most likely reason for soiling in an ARM patient would be incomplete emptying due to an ineffective bowel regimen. Essentially, for his whole life, constipation has not been worried about or managed. Also, given his malformation with a high rectum at the bladder neck, poor continence is expected. A full evaluation and workup should be completed to determine the best plan for the patient.

The mother, on her own and based on advice from a social network group, placed the patient on a daily rectal enema regimen of 350 mL of saline and 15 mL of glycerin. After being placed on these daily enemas, the child was clean and able to wear normal underwear. The patient and family are interested in transitioning from enemas to laxatives. 26

Figure 5.1  Contrast enema.

Case history

QUESTION 5.3 How do you interpret the findings on the contrast enema? Answer: Dilated ascending, transverse, and descending colon. Significantly distended rectum and distal sigmoid colon. Sigmoid redundancy.

The patient reported seven voluntary bowel movements and one stool accident in the last 24  hours. Five of the voluntary bowel movements were within 2 hours of each other.

QUESTION 5.6 What is your assessment of the X-ray?

QUESTION 5.4 Why do you think the colon looks this way? Answer: The dilation is due to years of constipation and inadequate colonic emptying.

QUESTION 5.5 According to the ARM continence predictor index, what is the patient’s potential for bowel control (refer to the figure “ARM Index” in the color insert)? Answer: His original malformation is a recto–bladder neck fistula (3 points), his conus terminates at L4 (tethered cord, underwent release) (2 points), and he has a sacral ratio of 0.66 (2 points). Therefore, the patient has poor potential for bowel control (7 points in total).

Despite the prediction of having a poor potential for continence, the patient and family desired to trial transitioning from rectal enemas to laxatives. The patient was started on senna, 45 mg once daily with 3 g of water soluble fiber twice a day (BID). Rectal enemas were stopped (Figure 5.2).

Answer: There is stool in the ascending and transverse colon. The descending colon and the rectum are empty.

QUESTION 5.7 Based on the X-ray and the stooling report from the patient, what regimen changes would you make? Answer: The bowel regimen changes were: ▪▪ Continue senna, 45 mg daily ▪▪ Increase water soluble fiber to 6 g BID

The laxative dose that the patient is on is properly emptying his colon of stool. The patient reports multiple bowel movements in a 24-hour period. The increase in water soluble fiber should work to consolidate the number of stools the patient is having (Figure 5.3).

Figure 5.3  Abdominal X-ray, Bowel Management Day 3.

Figure 5.2  Abdominal X-ray, Bowel Management Day 2.

On day 3 of bowel management the patient reported three small, soft voluntary bowel movements with three stool accidents in the last 24 hours. The patient had not 27

A patient with a good anatomic ARM repair, but poor potential for bowel control

had a bowel movement yet prior to the X-ray. His current bowel regimen was senna, 45 mg daily with water soluble fiber, 6 g BID.

QUESTION 5.8 What changes would you make to this patient’s bowel regimen based on the X-ray and report from the last 24 hours? A. Continue the same regimen B. Increase laxatives C. Decrease laxatives D. Increase fiber E. Decrease fiber Answer: The senna was increased to 60 mg once daily (Answer = B) due to stool accumulation throughout the colon on the X-ray. The increase in senna will also help the patient empty more effectively with the goal being one to two formed bowel movements per day.

The patient’s regimen was changed to senna, 60 mg daily with fiber, 6 g BID. The patient reported five soft, formed voluntary bowel movements with two stool accidents in the last 24 hours. The patient is having four stools in the morning; both accidents were in the afternoon. The abdominal X-ray from the day was clean.

QUESTION 5.9 What regimen changes can you make to decrease the number of stools the patient is having a day? A. Decrease laxatives B. Increase laxatives C. Add loperamide Answer: The patient’s previous regimen was 45 mg of senna, which was not enough to empty his colon, so you

28

do not want to decrease the laxative dose. The patient has a clean X-ray, an increased dose would cause more stooling and more accidents. Loperamide can be added to help quiet the colon (Answer = C), provide some bulk to the stool, and prevent accidents.

The patient’s regimen is changed to senna, 60 mg daily, fiber 6 g BID and loperamide 1 mg in the morning. He did not have a bowel movement for 22 hours after the loperamide, and he had increased stool in the colon on the X-ray. His loperamide dose was decreased to 0.5 mg in the morning. The regimen at the end of bowel management week was senna, 75 mg daily with fiber, 6 g BID. The loperamide was stopped because it caused too much bulk. The patient continued to struggle, having several voluntary stools a day and several accidents a day.

QUESTION 5.10 Other than loperamide, what can be done to decrease the number of stools the patient is having? Answer: A dose of water soluble fiber can be added during the day to add bulk and consolidate the stool to decrease the number of stools a day.

A dose of fiber was added in the middle of the day. The patient’s regimen was then senna, 75 mg daily with fiber 6 g in the morning and evening and 3 g in the afternoon. The patient continued to report multiple stools a day with multiple stool accidents a day. Despite several changes to both the laxative and fiber, a regimen was not found that effectively emptied the colon and produced one to two well-formed stools a day with no accidents. Essentially, we were unable to find the “sweet spot”—the combination of diet, fiber, and senna that produce one to two well-formed stools per day with control. The decision was made to return to rectal enemas.

Suggested readings

QUESTION 5.11 What else can be done for this patient? Answer: The option of a Malone appendicostomy can be offered to this patient to allow for an antegrade approach for

enemas. An antegrade enema option will allow the patient to be independent as he gets older (Figure 5.4).

Malone

Neo-Malone

Appendicostomy for antegrade flushes

Umbilical orifice

Figure 5.4  Malone appendicostomy.

KEY LEARNING POINTS 1. The patient and family’s goals and wishes need to be evaluated and taken into consideration when making decisions regarding bowel regimens. 2. Patients with poor potential for bowel control can potentially be successful on a laxative regimen and should be allowed to trial laxatives if desired. If they are not successful, a mechanical regimen is needed and a Malone offers a comfortable antegrade option which allows for independence. 3. Peristeen and a sacral nerve stimulator are also considerations for this patient which could tip them

SUGGESTED READINGS Lawal, T. A., Rangel, S. J., Bischoff, A., Pena, A., & Levitt, M. A. 2011. Laparoscopic-assisted malone appendicostomy in the management of fecal incontinence in children. Journal of Laparoendoscopic and Advanced Surgical Techniques, 5(21), 455–459.

over into continence. (This is the subject of a future chapter.) 4. An increased number of daily bowel movements on a laxative regimen can mean several things. It can mean that a patient is overstimulated, requiring a decrease in the laxative dose. It can also mean that the patient is understimulated and is stooling small amounts without fully emptying the colon. Additionally, it could mean that the stool needs to be bulked and consolidated, which can be achieved with water soluble fiber. An X-ray helps differentiate between the possibilities.

Rangel, S. J., Lawal, T. A., Bischoff, A., Chatoorgoon, K., Louden, E., Pena, A., & Levitt, M. A. 2011. The appendix as a conduit for antegrade continence enemas in patients with anorectal malformations: Lessons learned from 163 cases treated over 18 years. Journal of Pediatric Surgery, 46(6), 1236–1242.

29

6

A patient with a history of a cloacal malformation who needs colorectal, urological, and gynecological collaboration KRISTINA BOOTH

CASE HISTORY A 5-year-old female with a history of cloacal malformation, was previously repaired at an outside hospital and also had a tethered cord release as an infant. She presents to your clinic for bowel management and a multidisciplinary evaluation by colorectal, urology, and gynecology. This patient wears a diaper for both stool and urine. From the colorectal standpoint, she is taking polyethylene glycol 3350 17 g once daily. She has significant perineal skin breakdown due to frequent loose stools. From the urological perspective, the patient is incontinent of urine and denies feeling the sensation to void. She has a history of afebrile urinary tract infections every 6 months and is treated regularly by her primary care provider for these with oral antibiotics. The family is motivated for the patient to be clean of stool and dry of urine, and they report that the patient has never had a gynecological assessment.

▪▪ Cystoscopy showed a trabeculated bladder. The urethra was 4.2 cm in length and was catheterizable with an 8F coude catheter. ▪▪ Vaginoscopy showed a vagina with evidence of a previously resected vaginal septum and two normal appearing cervices. ▪▪ Lateral sacral ratio of 0.5. ▪▪ A contrast enema via the rectum is shown in Figure 6.1.

QUESTION 6.1 What initial workup would you recommend for this patient? Answer: Workup included: ▪▪ Examination under anesthesia, which showed a wellpositioned and properly sized anus.

30

Figure 6.1  Contrast enema.

Case history

QUESTION 6.2 What would be your colorectal plan for bowel management based on this information? A. No change to her bowel regimen B. Start an oral laxative C. Start an enema regimen Answer: C. Given the patient’s history of cloaca, tethered cord, and sacral ratio of 0.5, she has poor potential for bowel control. She will likely not be successfully continent for stool on an oral laxative regimen, as she has been treated with thus far. Therefore, she would benefit from a daily enema regimen, a mechanical program to empty her colon once per day. ▪▪ Urological evaluation included a renal ultrasound and voiding cystourethrogram as part of the video urodynamics. The renal ultrasound showed right Society of Fetal Urology (SFU) grade 3 hydronephrosis with a dilated ureter and left SFU grade 2 hydronephrosis. Video urodynamics are shown in Figures 6.2 and 6.3.

Figure 6.2  Video urodynamics imaging.

Figure 6.3  Urodynamics.

QUESTION 6.3 What is your interpretation of the video urodynamics? A. Normal urodynamics B. Smaller than expected bladder capacity, but low detrusor pressures with normal bladder compliance. Normal appearing bladder seen on the video portion with right grade 3 vesicoureteral reflux (VUR) C. Reduced bladder compliance, small bladder capacity and elevated detrusor pressures. Trabeculated bladder on video portion with right grade 3 VUR Answer: C. The urodynamics showed a smaller than expected bladder capacity. The patient’s estimated bladder capacity using the Koff formula [(age in years + 2) × 30 = bladder capacity in mLs] is 210 mL, her actual bladder capacity was 80 mL. There was also reduced bladder compliance with elevated detrusor pressures up to 63 cm H2O. Elevated detrusor pressure is defined as pressures over 40 cm H2O and may put a patient at risk for upper tract deterioration. On the video portion, the bladder appeared trabeculated and there was right grade 3 VUR. These video urodynamic findings are concerning for future upper urinary tract deterioration.

31

A patient with a history of a cloacal malformation who needs colorectal, urological, and gynecological collaboration

QUESTION 6.4

QUESTION 6.7

Who might you also consult given these urodynamic findings?

Why do we wait at least 3 months after a tethered cord release to repeat urodynamics testing?

Answer: Given the urodynamic findings of reduced bladder compliance and high detrusor pressures, neurosurgery was consulted. The neurosurgical team often uses urodynamic findings to help guide their decision-making regarding a tethered cord. Specifically, neurosurgery is looking for bladder changes or bladder deterioration noted on the urodynamics. A repeat spinal MRI was also completed on this patient that showed concern for retethering of the spinal cord. Given the MRI and urodynamic findings, the neurosurgery team recommended surgery to re-release the tethered cord.

Answer: There is a period of spinal shock initially after the tethered cord release, so to assess long term bladder function it is best to wait at least 3 months to reassess the bladder dynamics.

QUESTION 6.5 What was the goal of the tethered cord re-release? Answer: The goal of the tethered cord re-release was to prevent continued deterioration of bladder/bowel function and nerve function.

QUESTION 6.6 While awaiting the tethered cord surgery, what would you recommend for the colorectal and urological plans? Answer: Colorectal: The patient was started on a daily rectal enema program. She completed bowel management with a regimen of 400 mL of saline and 25 mL of glycerin. On this regimen she was having no fecal soiling, the enema time was 1.5 hours, and her abdominal X-rays showed no significant stool accumulation after her enema. Urological: Given the urodynamics findings with reduced bladder compliance and elevated detrusor pressures, the patient was started on clean intermittent catheterization every 3–4 hours to empty the bladder and an anticholinergic medication to help improve bladder filling pressures. Our plan is to repeat a renal ultrasound and urodynamics in 3 months after the retethered cord release.

32

Follow up (3 months later): Colorectal: The patient continued with rectal enemas. Her regimen is 400 mL of saline and 30 mL of glycerin. She started to have minimal soiling in between enemas each day and began to have a longer flush time of 2.5 hours. We increased the strength of her enema regimen to try to improve colonic emptying, but the patient developed abdominal cramping and did not tolerate this increased regimen.

QUESTION 6.8 What could you offer this patient? A. Antegrade option, such as Malone appendicostomy B. Antegrade option and colon resection C. No change to bowel regimen Answer: B. We discussed a Malone appendicostomy and the potential for a colon resection given the lengthy enema time and the patient not tolerating a more concentrated enema regimen. One can anticipate that a sigmoid resection will improve the flush time by about 1 hour. Urological: Repeat urological testing 3 months after starting catheterizations and while on an anticholinergic showed improving right SFU grade 2 hydronephrosis and dilated ureter and stable left SFU grade 2 hydronephrosis. Her urodynamics showed increased bladder capacity of 190 mL (expected capacity of 210 mL) and improved bladder compliance with a maximum detrusor pressure of 25 cm H2O (previously had maximum detrusor pressure of 63 cm H2O). It is important to note the improvement to the upper urinary tract is directly a result of improved bladder management. Aggressive and proactive treatment to ensure bladder emptying and safe bladder filling pressures can prevent upper tract changes and chronic kidney disease. The patient was not tolerating urethral catheterizations well and catheterizations proved to be difficult. The patient was dry in between catheterizations when catheterized every 4 hours.

Case history

QUESTION 6.9 How would you proceed?

Options for the appendix depend on its length ≤5 cm

5–7 cm

7 cm

Appendix = Malone Small bowel = Mitrofanoff

Appendix = Mitrofanoff Cecum = Neomalone

Proximal appendix = Malone Distal appendix = Mitrofanoff

A. Appendicovesicostomy B. Appendicovesicostomy and bladder augmentation C. Appendicovesicostomy with bladderneck procedure to increase outlet resistance

Intraoperative: The patient underwent surgery and had a Malone appendicostomy and appendicovesicostomy using the split appendix technique (Figures 6.4 and 6.5)

MM CM

Answer: A. The urological plan was for an appendicovesicostomy, given the patient was not tolerating urethral catheterizations well. Since the patient’s repeat urodynamics showed improved bladder capacity and improved bladder dynamics, there was no need to consider a bladder augmentation. The patient was also dry in between catheterizations, thus she did not require any bladder outlet procedure. After a collaborative discussion with the colorectal, urological, and gynecological teams, the plan we formulated was to perform a Malone appendicostomy and appendicovesicostomy using a split appendix technique and sigmoid resection. If the urology team felt a bladder augmentation was necessary, the sigmoid colon that is already being resected could be used for the bladder augmentation. The gynecology team also planned to perform an inspection of Mullerian structures at the time of this reconstructive surgery, as this is an ideal time to confirm gynecological anatomy and allow for proper planning of menstruation and future obstetrical potential.

0

1

2

3

4

5

6

7

8

9

10 11 12

Figure 6.5  Options for the appendix.

and a sigmoid colon resection. The gynecology team also did an inspection that showed normal bilateral fallopian tubes and ovaries and two uterine horns. Postoperative follow up and plan: The patient did well, postoperatively. Colorectal: The patient’s antegrade flush was adjusted to 400 mL of saline and 10 mL of glycerin. For the first month this was given as 200 mL of saline and 5 mL of glycerin twice a day before being combined into a single flush. Her flush time was reduced to 45 minutes and she is not having any soiling in between her flushes. Urological: The patient is catheterized per appendicovesicostomy every 3–4 hours and doing well. She is on an anticholinergic daily and remains dry in between catheterizations. She has not had any urinary tract infections and is scheduled for routine renal ultrasounds for follow up to monitor for upper urinary tract changes. Gynecological: The patient will have a pelvic ultrasound 6 months after thelarche. Normal menses and good obstetric potential are anticipated.

KEY LEARNING POINTS

Figure 6.4  Split Malone appendicostomy.

1. It is important to do an initial thorough assessment when evaluating a patient with complex ARM. This initial assessment should include a colorectal evaluation, urological evaluation, and gynecological evaluation. 2. For patients with a history of tethered cord, it is important to have a neurosurgical assessment if there are urological symptoms. It is helpful to have both a repeat spinal MRI and urodynamics to help guide the neurosurgeon’s decision-making.

33

A patient with a history of a cloacal malformation who needs colorectal, urological, and gynecological collaboration

3. For patients who have completed bowel management with enemas who are unable to tolerate the amount of stimulants required to empty the colon or have a longer flush time, a colon resection (of the sigmoid colon) should be considered. 4. Patients with complex ARM should be cared for by a multidisciplinary team. Often, surgeries are able to be coordinated together to eliminate additional anesthetics with this team approach.

SUGGESTED READINGS Eradi, B., Hamrick, M., Bishoff, A., Frischer, J. S., Helmrath, M., Hall, J., Pena, A., & Levitt, M. A. 2013. The role of a colon resection in combination with a Malone appendicostomy as part of a bowel management program for the treatment of fecal incontinence. Journal of Pediatric Surgery, 48(11), 2296–300.

34

Pradhan, S., Vilanova-Sanchez, A., McCracker, K. A., Reck, C. A., Halleran, D. R., Wood, R. J., Levitt, M. A., & Hewitt, G. D. 2018 November. The Mullerian Black Box: Predicting and defining Mullerian anatomy in patients with cloacal abnormalities and the need for longitudinal assessment. Journal of Pediatric Surgery, 53(11), 2164–2169. Koff, S. A. 1983 March. Estimating bladder capacity in children. Journal of Urology, 21(3), 248. Vanderbrink, B. A., Levitt, M. A., Defoor, W. R., & Alam, S. 2014 April. Creation of an appendicovesicostomy Mitrofanoff from a preexisting appendicocecostomy utilizing the split appendix technique. Journal of Pediatric Surgery, 49(4), 656–659. Vilanova- Sanchez, A., Halleran, D. R., Reck-Burneo, C. A. et al. 2019 March. A descriptive model for a multidisciplinary unit for colorectal and pelvic malformations. Journal of Pediatric Surgery, 54(3), 479–485.

7

A young adult with prior surgery for an anorectal malformation (ARM) with fecal incontinence ONNALISA NASH

CASE HISTORY A 25-year-old female with a history of a complex cloaca comes to your clinic. She underwent cloacal reconstruction with a posterior sagittal anorectovaginourethroplasty (PSARVUP) as a baby and subsequently underwent a redo of her repair with partial vaginal replacement with rectum several years later due to an inadequate perineal body and introital stenosis. She underwent a Malone appendicostomy at age 5 to provide for antegrade access for flushes. Over time, she continued to require more concentrated and voluminous daily flushes to empty. The flush volume and additives were being increased by the nursing team regularly and the flush was taking longer than an hour and a half. Her Malone leaks stool 1 to 2 times per week. She presents to you for evaluation for help with improving her bowel management regimen. Her Malone flush currently is 200 mL saline, 30 mL glycerin, and 30 mL Castile soap, then 200 mL saline 10 minutes after administration. She also takes Colace 100 mg daily. She continues to experience stool accidents daily and prolonged emptying. Her urological history includes chronic kidney disease (CKD) stage 1–2, bladder and kidney stones, hydronephrosis, and bilateral vesicoureteral reflux. She catheterizes per urethra and is dry between cathing.

D. Contrast enema E. Sitz marker study Answer: A and D. A is important to evaluate the position and anatomy of her anal opening, as well as the location and quality of her urethra and vagina. D is important to evaluate the anatomy of her colon including location, length, and diameter.

EVALUATION • EUA/cysto/vaginoscopy: Anus well-positioned and without stricture; the cystoscopy and vaginoscopy were unremarkable • Magnetic resonance imaging (MRI) of the spine revealed sacral dysplasia and no tethered cord • Contrast enema is shown in Figure 7.1

QUESTION 7.1 Which two evaluations from the following list would be pertinent to initially assess this patient? A. Examination under anesthesia (EUA)/cysto/ vaginoscopy B. Anorectal manometry C. Colonic manometry

Figure 7.1  Final image of contrast enema.

35

A young adult with prior surgery for an ARM with fecal incontinence

• Sacral ratio: AP 0.3 and lateral 0.39 (Figure 7.2 and refer to the figure “ARM Index” in the color insert)

QUESTION 7.4 After assessing this contrast study how would you describe the colon?

(a)

Answer: The contrast study reveals a narrowed segment of the distal aspect of the descending colon. Note: It is important to assess the anatomy in order to troubleshoot any problematic flush by simulating the flush using the Malone and performing an antegrade study.

QUESTION 7.5 What do you think this narrowing represents? (b)

Answer: This is likely due to ischemia at the area of the previous colostomy closure and/or area of colonic vaginal interposition.

QUESTION 7.6 Knowing this segment is narrowed, what would your next step be? A. Increase strength of flush to help improve emptying B. Surgery to resect the narrowed segment C. Add loperamide to slow transit of distal colon D. Colonic manometry to assess motility of distal colon Figure 7.2  (a) AP X-ray image of sacrum and (b) lateral image of sacrum.

QUESTION 7.2 Are these sacral ratios poor, fair, or good? Answer: Based on the values of sacral ratio poor (0.7), this sacral ratio would be considered poor, and represents significant caudal regression.

QUESTION 7.3 Considering the ARM index as a predictor of continence, what would you predict is this patient’s probability for continence? Answer: The patient has a poor to intermediate probability for continence, due to a high type of malformation, normal spine, and poor sacrum.

36

Answer: B (surgical intervention). ▪▪ We decided to perform an exploratory laparotomy, colonic resection of the narrow-left colon, and Malone revision due to leaking stool from the Malone. Of note, the rectal pull-through was widely patent. The area of narrowing began at the distal left colon and reached to the peritoneal reflection. This was the area we resected. Her surgery went well. The left colon was quite narrow but a healthy transverse colon was successfully anastomosed to the neo rectum, which was normal caliber.

Postoperatively, the patient was on twice daily flushes for 1 month then was transitioned to Malone flushes with saline 300 mL and glycerin 20 mL. Her flushes were initially improved after surgery, but over time the length of flush increased. She contacted our center with concerns that the flush was taking over 2 hours. An abdominal X-ray was obtained (Figure 7.3).

Evaluation

The X-ray now shows a clean colon. The flushes are working and the patient is clean.

Figure 7.3  Abdominal X-ray.

QUESTION 7.7 In a patient with the X-ray shown in Figure 7.3 and extended flush time, what change should be made to the flush? A. Increase saline B. Increase glycerin C. Decrease glycerin D. Decrease saline Answer: A and B. Change Malone flushes to saline 350 mL and glycerin 30 mL to increase volume and strength due to stool in descending colon. The current flush was not emptying the colon well.

Her follow up X-ray is shown in Figure 7.4. The patient is now doing well with no soiling and the flushes are taking around 1 hour.

Figure 7.4  Abdominal X-ray after flush adjustment.

KEY LEARNING POINTS 1. The initial workup is invaluable, especially the EUA and contrast imaging. Simulating the flush with an antegrade contrast study is vital. 2. A long flush time can mean a few things, including: (1) the flush is not a large enough volume to make it through the colon; (2) the flush is too strong and is causing continued contractions of the colon; (3) the flush is not strong enough and is not emptying the colon; (4) there is an anatomic blockage impeding the flush; and/or (5) there is reflux of the flush into the terminal ileum (discussed in a subsequent case). Abdominal X-rays help to clarify which case you are dealing with.

37

8

A patient with an anorectal malformation (ARM) with fecal incontinence who is a candidate for a sacral nerve stimulator (SNS) CATHERINE TRIMBLE

CASE HISTORY A 12-year-old male presents to your clinic with a history of an anorectal malformation (ARM). The original malformation is unknown. He was initially managed with a colostomy and underwent a posterior sagittal anorectoplasty (PSARP) and colostomy closure in his first year of life. He subsequently underwent two reoperations for anterior mislocation of his

anus and a stricture, as well as a Malone appendicostomy for antegrade enema administration. He participated in a bowel management program to determine the ideal flush regimen. His current antegrade flush regimen is saline 500 mL, glycerin 30 mL, and Castile soap 5.5 mL. He is clean on this regimen with only occasional stool accidents (one to two per month). The patient is interested in transitioning from antegrade flushes to laxatives.

QUESTION 8.1 Which of the following diagnostic tests would you want to obtain to predict continence potential?

E. Sacral X-ray F. Colonic motility

A. Abdominal X-ray B. MRI spine C. MRI pelvis D. Contrast enema

Answer: B and E.

Type of malformation

The potential to be continent is based on three factors: the type of anorectal malformation that a patient has, the quality

Quality of sacrum

Figure 8.1  Predictors of continence in ARM.

38

Status of spine

Evaluation

of the patient’s spine, and the quality of their sacrum (ARM continence index) (Figure 8.1). The type of malformation should be known from the initial imaging and operative findings at the time of the initial ARM repair. The sacral X-ray will provide information regarding the

quality of the sacrum and objectively assess the degree of caudal regression. An MRI of the spine provides information on the quality of the patient’s spine and assesses for tethered cord.

EVALUATION • Original malformation is unknown • Sacral ratio AP: 0.64, lateral: 0.84 • Normal MRI of the spine, no evidence of tethered cord Examination of the perineum revealed an anus located within the sphincter complex, no prolapse or stricture, and good sphincter contractions were noted.

QUESTION 8.2 Using the ARM continence predictor index, what is the patient’s potential for bowel control (refer to the figure “ARM Index” in the color insert)? Answer: The MRI was normal (1 point). The lateral sacral ratio was 0.84 (1 point). The original malformation is unknown, but the patient appeared to have had a good repair with good sphincter muscles. Total points: 3–5. Therefore, the patient has a good potential for bowel control. He should be successful on laxatives and no longer need antegrade flushes.

The plan was to have him participate in a bowel management week to transition from antegrade enemas to laxatives.

QUESTION 8.3 What diagnostic test would you want to obtain prior to beginning bowel management? A. Colonic motility B. Contrast enema C. Abdominal X-ray Answer: B. The contrast enema allows the provider to fully assess the colon, noting the colonic anatomy and degree of colonic distention. It provides valuable information that helps guide the choice for the starting bowel regimen (Figure 8.2).

Figure 8.2  Contrast enema, Bowel Management Day 1.

QUESTION 8.4 What is your assessment of the contrast enema? What would your starting bowel regimen be? Answer: The patient has a somewhat dilated colon without redundancy. He should be able to empty well, but will most likely need stimulant laxatives.

Based on the contrast study, it was determined that the patient should be started on a moderate dose of stimulant laxatives. Water-soluble fiber was also started to add bulk and form to the stool. The patient was started on senna 45 mg and 3 g of water-soluble fiber twice a day (BID) (Figure 8.3). On bowel management day 4, the patient’s regimen is senna 37.5 mg (2.5 squares) with water-soluble fiber 4 g BID. The patient reports several bouts of diarrhea and soiling overnight. 39

A patient with an ARM with fecal incontinence who is a candidate for a SNS

Regimen is 22.5 mg (1.5 squares) with 5 g of watersoluble fiber BID. Patient reports eight stools in 24 hours with one accident overnight.

QUESTION 8.6 How would you manage this patient moving forward? A. Continue current dose of senna and fiber B. Continue to adjust senna and fiber C. Resume previous regimen of Malone flushes D. Resume previous regimen of Malone flushes and consider a sacral nerve stimulator (SNS) Figure 8.3  Abdominal X-ray, Bowel Management Day 4.

QUESTION 8.5 Based on the X-ray and the patient’s report, what regimen changes would you make? A. Increase laxatives B. Decrease laxatives C. Increase fiber D. Decrease fiber Answer: B. The patient has stool in the ascending colon and has a report of several stools with soiling accidents but mostly controlled bowel movements. This could indicate that the laxative dose is too high and the patient is being overstimulated. The decision was made to decrease the laxative dose.

The regimen was changed to senna 30 mg once daily with 4 g of water-soluble fiber BID (Figure 8.4).

Answer: D. The laxative trial was not successful. For now, going back to flushes makes the most sense to get the patient clean again. The SNS can help patients who have done well on an enema regimen, but have been only partially successful transitioning to laxatives. The SNS can help by improving anal canal sensation and/or sphincter control.

The patient continued to have multiple stools a day with soiling despite several regimen changes. The patient and the family believed that the patient’s quality of life was better while on antegrade flushes. The decision was made to resume the patient’s previous bowel regimen. Essentially, with more laxatives, the colon empties but the patient does not have adequate bowel control. When the laxative dose is lowered, then the patient does not stool, and the “sweet spot” whereby the right amount of laxative and fiber provide the stool consistency needed for continence cannot be achieved. Therefore, the patient needs a mechanical regimen. It is in just such a case that SNS could have a valuable role.

QUESTION 8.7 What information and testing would you need before proceeding with the SNS?

Figure 8.4  Abdominal X-ray, Bowel Management Day 7.

40

Answer: Colonic manometry (CMAN) would be needed to assess for colonic dysmotility and in this case showed normal motility throughout most of the colon with lower motor activity in the distal third of the colon. Anorectal manometry (AMAN) is vital to assess baseline rectal sensation and the patient’s ability to squeeze, and in this case showed an intact rectoanal inhibitory reflex

Suggested reading

(RAIR), normal anal sphincter pressure, and good squeeze pressure. Therefore, no outlet procedures are needed such as botulinum toxin or biofeedback. Sacral X-rays, to understand the degree of caudal regression and for SNS procedure planning, were already obtained. An MRI of the pelvis and spine were already obtained to assess for any spinal anomalies or indications for a redo PSARP, such as a remnant of the original fistula.

The decision was made to trial an SNS and a temporary lead was placed. An AMAN was repeated 4 days later, which showed improved rectal sensation. The patient reported that he was able to sense stool in his rectum and that he was able to have controlled bowel movements between his flushes. The patient did well with the temporary SNS and a permanent SNS was inserted 1 month later. The patient then returned for bowel management to transition from rectal enemas to laxatives. A repeat contrast enema, now 1 year after the previous one, was obtained (Figure 8.5).

This X-ray was obtained after starting senna and holding flushes (Figure 8.6). Day 7 of bowel management week: The regimen was senna 75 mg once daily with 1 g of water-soluble fiber TID. The patient reported one to two formed bowel movements a day with no soiling accidents.

Figure 8.6  Abdominal X-ray, Bowel Management Day 7.

KEY LEARNING POINTS 1. Despite having a good potential for bowel control, the patient was initially unable to transition from antegrade enemas to laxatives. 2. Patients with ARM may benefit from an SNS if they desire to be continent on laxatives and have potential for continence. SNS can “tip them over the edge” into continence. It has also been shown to improve the antegrade flush, and is known to improve urinary/bladder function as well. 3. It is important to understand the patient’s and family’s goals when discussing bowel management and future options.

Figure 8.5  Contrast enema, Bowel Management Day 1.

Contrast enema via the Malone prior to starting bowel management showed improved peristalsis in the left colon as indicated by haustral markings. He was started on senna 45 mg and water-soluble fiber 2 g three times a day (TID).

SUGGESTED READING Sulkowski, J. P., Nacion, K. M., Deans, K. J., Minneci, P. C., Levitt, M. A., Mousa, H. M., Alpert, S. A. & Teich, S. 2015. Sacral nerve stimulation: A promising therapy for fecal and urinary incontinence and constipation in children. Journal of Pediatric Surgery, 50(10), 1644–1647. 41

PART

III

HIRSCHSPRUNG DISEASE

INTRODUCTION Stacie Leeper Hirschsprung disease (HD) is a congenital malformation in which the nerves (ganglion cells) in the colon do not form properly. The incidence of HD is approximately 1 in 5,000 live births. These nerves are responsible for peristalsis; therefore, patients with HD develop functional intestinal obstruction starting at the level at which the nerves failed to migrate. The bowel downstream from the ganglion cells is unable to relax, therefore this distal segment remains contracted leading to a functional obstruction. The level of aganglionosis can occur anywhere in the colon and, in rare cases, even involve the entire colon and small bowel (total intestinal HD). The most common type typically affects the left colon or sigmoid colon. HD usually presents in the newborn period when the infant does not pass meconium in the first 48 hours of life. The infant with HD may also have abdominal distention, explosive, foul-smelling stools, bilious emesis, feeding intolerance, or failure to thrive. This presentation of enterocolitis results from the stasis of stool leading to bacterial overgrowth. The bacterial overgrowth can then lead to translocation (the process by which bacteria crosses into the bloodstream through cracks in the intestinal mucosa) which can make the patient very ill. Older children diagnosed with HD often present with chronic constipation. If HD is suspected in the newborn period, rectal irrigations should be initiated to decompress the colon. During a rectal irrigation, a soft tube is passed into the rectum and the colon is gently “washed” with small volumes of saline. The goal of rectal irrigation is to remove stool and gas from the bowel and prevent stasis of stool in the colon. Once the colon is adequately decompressed, contrast enema should be performed. The classic finding of HD on contrast enema is a transition zone between the normal and aganglionic bowel (see Figure III.1).

Hirschsprung disease

Figure III.1  Decompressed bowel in the rectosigmoid, dilated bowel up to the left colon, the transition zone is suspected to be in the proximal sigmoid.

Another helpful diagnostic tool for children over 1 year of age is the use of anorectal manometry to evaluate for recto-anal inhibitory reflex (RAIR). The RAIR is reflexive relaxation of the internal anal sphincter in response to rectal dilation and is found to be absent in children with HD. Anorectal manometry is useful in the evaluation of an older child who suffers from chronic constipation. In this case, if the RAIR is present, the diagnosis of HD can be ruled out, therefore eliminating the need for rectal biopsy. If RAIR is absent, HD must be confirmed with rectal biopsy. If the biopsy is normal the patient may have internal sphincter achalasia, which will be discussed in a subsequent chapter. The gold standard for definitive diagnosis is a rectal biopsy that demonstrates the absence of ganglion cells in the submucosa as well as the presence of hypertrophic nerves. In the newborn period, a suction rectal biopsy can be done, however full-thickness biopsy is necessary in children over 1 year of age. Once the diagnosis has been made, the definitive treatment is surgical. The goal of surgical management is to remove the aganglionic segment of the bowel and bring the normally innervated segment down to the anus. The anal sphincters and anal canal must be preserved in order for the child to achieve future continence. The three most commonly performed 44

Figure III.2  Soave technique.

operations for HD are the Soave, Swenson and Duhamel procedures (see Figures III.2–III.4). The Soave pullthrough removes mucosa and submucosa of the rectum, and the ganglionated bowel is pulled down and placed within a cuff of the aganglionic muscle. The Swenson

Figure III.3  Swenson technique.

Hirschsprung disease

Figure III.4  Duhamel technique.

pull-through removes the entire aganglionic segment of the bowel, with an end to end anastomosis above the anal sphincter. In the Duhamel procedure, the normal colon is brought down through the plane between the rectum and the sacrum. The two walls are joined to create a new lumen that is aganglionic anteriorly and normally innervated on the posterior side. In all cases, the anal canal should be carefully preserved. Therefore, if surgery is done well, the abnormal smooth muscle (internal) sphincter remains intact, which can have clinical implications later. The complications that can occur after pull-through surgery can typically be placed in one of the two following categories: (1) obstructive or (2) soiling. Obstructive symptoms include abdominal distention, vomiting or episodes of enterocolitis, similar to a newly diagnosed case of HD. Children with enterocolitis in the postoperative period present with abdominal distention, fever, vomiting, feeding intolerance and foul-smelling explosive stool. Chronic enterocolitis and obstructive symptoms can lead to failure to thrive. Treatment for enterocolitis includes rectal irrigations, hydration and metronidazole (Flagyl). Obstructive symptoms can be caused by the mechanical obstruction from a stricture, a twist in the pull-through,

an obstructing Soave cuff, a transition zone pull-through or a large Duhamel pouch. If none of these are present, the non-relaxing sphincter can be the problem. The second category of complication following a pull-through procedure is soiling. Soiling can be due to damage to the sphincter or dentate line during the original operation. A surgeon must be meticulous in protecting these structures so that a child is not left with injured continence mechanisms. Soiling can also occur with intact sphincters and dentate line when the colon either moves too quickly (hypermotility) or too slowly (hypomotility). This can be treated medically by creating a good bowel movement pattern with the goal of one to two soft, formed stools per day, either by speeding up or slowing down the stooling. Children with HD naturally have “tight” anal sphincters that do not relax normally. While they are still young, many may need botulinum toxin injection into the anal canal until they are old enough to learn how to overcome that sphincter on their own which will allow them to empty their colon successfully. Botulinum toxin can be repeated as frequently as every 3 months as necessary.

SUGGESTED READINGS Frykman, P. K. & Short, S. S. 2012. Hirschsprungassociated enterocolitis: Prevention and therapy. Semin Pediatr Surg. 21(4),328–35. Koivusalo, A. I., Pakarinen, M. P., & Rintala, R. J. 2009. Botox injection treatment for anal outlet obstruction in patients with internal anal sphincter achalasia and Hirschsprung’s disease. Pediatric Surgery International 25(10),873–876. Langer, J. C., Rollins, M. D., Levitt, M. et  al. 2017. Guidelines for the management of postoperative obstructive symptoms in children with Hirschsprung disease. Pediatric Surgery International 33(5),523–526. Levitt, M. A., Dickie, B., & Pena, A. 2010. Evaluation and treatment of the patient with Hirschsprung disease who is not doing well after a pull-through procedure. Semin Pediatric Surgery 19, 146–53. Levitt, M. A., Dickie, B., & Pena, A. 2012 Nov. The Hirschsprung patient who is soiling after what was considered a “successful” pull-through. Semin Pediatric Surgery 21(4), 344–53.

45

9

A patient with good surgical anatomy and hypomotility after a Hirschsprung pull-through LEAH MOORE

CASE HISTORY A 3-year-old female with Hirschsprung disease and significant developmental delay presents with persistent constipation and overflow soiling. Originally, the patient was diagnosed at 2 years of age with Hirschsprung disease with a transition zone in the rectosigmoid and underwent a primary Swenson pullthrough. The X-ray done on her first follow-up visit 1 month after her pull-through is shown in Figure 9.1.

Figure 9.1  Abdominal X-ray at the one month visit.

QUESTION 9.2 What would be your recommended plan? Answer: The patient was advised to irrigate for any distention. No stooling medications were recommended.

The patient returned at 2 months and her mother reported that the patient was feeling well, eating well, and had resumed all normal activities. She was having three loose bowel movements a day and there had been no need for irrigations. An X-ray was performed (shown in Figure 9.2).

Figure 9.2  Abdomal X-ray.

QUESTION 9.1

QUESTION 9.3

What does this X-ray show?

What does this X-ray show?

Answer: No colonic distention post–pull-through, no retention of stool.

Answer: Moderate retained stool throughout the colon and a mildly dilated pull-through.

46

Water-soluble fiber

The patient returned for their scheduled 3-month clinic visit. An abdominal X-ray was performed (Figure 9.3).

QUESTION 9.4 What would be your recommended treatment? Answer: The X-ray shows retention of stool and gas. We were concerned about three things: (1) an anastomotic stricture; (2) obstruction from her non-relaxing sphincters; and (3) a slow-moving colon in need of laxatives. Because of this, the patient underwent an exam under anesthesia to confirm that there was no stricture (and none was found). The dentate line and sphincter complex were both intact. Botulinum toxin was administered into the internal sphincter to paralyze the non-relaxing internal anal sphincters, with the idea that as the botulinum toxin effect wears off over several months the child would “learn” to stool on their own, by pushing to overcome the non-relaxing sphincters.

QUESTION 9.5 What would be your plan now? Answer: To treat a slow-moving colon, once we confirmed there was no anatomic problem with the pull-through, we started the patient on 30 mg of senna daily to help to stimulate the bowel to move. In order to help consolidate and bulk the stool, we added 3 grams of water-soluble fiber twice per day and waited for the botulinum toxin to start to work.

WATER-SOLUBLE FIBER

Figure 9.3  Abdominal X-ray.

QUESTION 9.6 Why did we start senna? Answer: The patient’s X-ray showed a moderate amount of stool burden along with mild dilatation of the pullthrough. We concluded that the patient was not emptying effectively and needed the help of a stimulant laxative.

Refer to Table 9.1. Table 9.1  Water solube fiber options. Type Pectin (Sure-Jell)

Citrucel (methylcellulose)

Dosage and use

Where to find it

1 tablespoon = 2 g of fiber

Found in the grocery store in the jelly/canning section or online at www. pacificpectin.com. Get the sugar-free version.

Powder

Found in the pharmacy section of the store or online at www.citrucel.com. You can use the generic or the brand name. Get the sugar-free version.

1 tablespoon = 2 g of fiber Capsule 2 capsules = 1 g of fiber

Metamucil (psyllium husk)

Powder

Nutrisource (guar gum)

1 tablespoon (scoop) = 3 g of fiber (Can be sprinkled on food or mixed in drinks.)

1 teaspoon = 2 g of fiber Capsule 5 capsules = 2 g of fiber Wafer 1 packet (2 wafers) = 3 g of fiber

Found in the pharmacy section of the store or online at www.metamucil.com. Get the sugar-free version.

Found in the pharmacy section of the store, online, or through homecare companies.

47

A patient with good surgical anatomy and hypomotility after a Hirschsprung pull-through

QUESTION 9.7

QUESTION 9.9

What does this X-ray show?

What is the rationale behind adding water-soluble fiber to the stimulant laxative?

Answer: There is formed stool throughout in the ascending, transverse, and sigmoid segments. There is mild distention of a stool-filled rectum.

Stooling report: • Senna 30 mg once daily; the mother had not been giving the recommended water-soluble fiber, instead choosing to give “high fiber foods” • Three to four loose, runny bowel movements daily • No irrigations have been done due to the patient not having any distension • No fever, foul-smelling stool, straining to stool, cramping, nausea, or vomiting

QUESTION 9.8 Based on this report, what would you suggest? Answer: We increased the senna to 45 mg once daily and encouraged the mother to give the recommended 6 g of supplemental water-soluble fiber twice per day. The family was to send a stooling report in 1 week. The plan was for them to follow up in clinic in 2 months with an X-ray prior.

Most recently, the mother reports difficulty with getting the child to take the fiber regimen. The patient is having 3–4 loose bowel movements per day. The mother reports no concerns for any obstructive symptoms. We encouraged her to continue to incorporate the fiber regimen into the diet as much as possible.

48

Answer: Stimulant laxatives can cause watery stools if not given with a water-soluble fiber. We use a water-soluble fiber to help absorb the water in the stool and create bulk to the stool.

KEY LEARNING POINTS 1. Laxatives can only be effective in this case because the patient has an anatomically correct pull-through and has the capacity to achieve voluntary bowel movements. 2. Starting laxatives are sometimes needed to help effectively empty the colon and help prevent colonic dilation resulting from stool retention. 3. Water-soluble fiber is also needed in conjunction with the laxative therapy to help bulk and create good consistency of stool. The key to success is to obtain a good bowel movement pattern; that is, one to two well-formed stools per day. 4. Compliance with the fiber regimen can be a challenge particularly in children with a diagnosis of developmental delay. 5. Botulinum toxin is used to paralyze (force relaxation) of the inherently non-relaxing sphincters of patients with Hirschsprung disease. This is only given to patients with an intact dentate line.

10

A patient with good surgical anatomy and hypermotility after a redo pull-through for Hirschsprung disease LEAH MOORE

CASE HISTORY A 3-year-old male with a history of Hirschsprung disease underwent a colostomy creation as a newborn and then a pull-through of his colostomy. His parents have had to do daily rectal irrigations for distention and the lack of spontaneous bowel movements. The patient also had three enterocolitis episodes requiring hospital admission post–pull-through.

Answer: Exam under anesthesia, rectal biopsy, and contrast enema were planned as part of his initial evaluation. The exam under anesthesia was to assess anatomy and the previous surgery. Rectal biopsies were needed to assess for the presence of ganglion cells and to check whether this was a transition zone pull-through. A contrast enema was performed to evaluate the pullthrough (Figure 10.1).

QUESTION 10.1

QUESTION 10.2

What would you do for an evaluation of these symptoms?

What do you see? Answer: The image demonstrates that the colon pullthrough comes down the right pelvis. There is very little colon (essentially, he only has his right colon) and mild dilation of the pull-through. The bowel opacified by the contrast on the left side is all small bowel.

An exam under anesthesia revealed no twist or stricture. The rectal biopsy showed an absence of ganglion cells and there were hypertrophic nerves.

QUESTION 10.3 What is your interpretation of those results and what would be your plan be, given these findings?

Figure 10.1  Contrast enema.

Answer: These findings are consistent with a transition zone pull-through. Based on these results and the obstructive symptoms, we performed a redo transanal

49

A patient with good surgical anatomy and hypermotility after a redo pull-through for Hirschsprung disease

pull-through for retained transition zone. The ganglionic bowel was found 6 cm higher in the colon. Thereafter, we planned to have the patient return in 1 month for an examination under anesthesia to assess the anastomosis.

One month after the redo, the patient’s mother reported 9 to 10 stools per day in the weeks following surgery, resulting in extensive perineal skin breakdown. There was no further distention or any obstructive symptoms.

QUESTION 10.7 Based on patient’s report and X-ray assessment, would this patient be hypermotile or hypomotile? A. Hypermotile B. Hypomotile Answer: Hypermotile. We concluded this based on the mother’s stooling reports of 9 to 10 liquid bowel movements (BMs) per day and no stool seen on X-ray image. He also has a severe perianal rash.

QUESTION 10.4 Do you think the redo was successful?

QUESTION 10.8

Answer: Yes—his obstructive symptoms have been eliminated. Now he is suffering from loose stools, which can be medically managed. And, this should be anticipated given how little colon he has.

Given the conclusion of hypermotility, what would your plan be?

QUESTION 10.5 What would your plan of action be? Answer: Obtain an abdominal image to assess stool burden. The abdominal X-ray is shown in Figure 10.2.

Answer: The plan was to begin adding water-soluble fiber to consolidate and bulk his stool, and decrease his frequency to a goal of two to three BMs per day. This also will improve the skin breakdown.

The patient presented for a planned 1-month exam under anesthesia. X-ray prior to exam under anesthesia is shown in Figure 10.3.

Figure 10.2  Abdominal X-ray. Figure 10.3  Abdominal X-ray.

QUESTION 10.6

QUESTION 10.9

What do you see?

What do you see?

Answer: The X-ray image showed no stool, and no gaseous distension.

Answer: There is mild to moderate formed stool in the rectum.

50

Case history

The patient underwent a planned 1-month exam under anesthesia. The anal canal and sphincter complex were intact. The anastomosis had healed well with no stricture.

No Botulism toxin was given. It was clear we were now successfully bulking the stool.

QUESTION 10.10 What would your plan be now? Answer: The plan was to increase the amount of fiber based on the stooling reports of liquid stools. The goal is to

achieve more stool bulk. Education on a constipating diet was provided and recommended (Tables 10.1 and 10.2).

Table 10.1  Constipating diet phase 1 Phase 1 Food group

Food recommendations

Milk Vegetables

Plain rice milk None

Fruits Starch, grains

Applesauce, apples (without skin), bananas White flour, refined flour Bread, crackers, pasta and noodles, white rice, white potatoes (without skin),dry cereals Baked/broiled/grilled meats, poultry or fish, lean deli meats, eggs Nonstick spray, nonfat butter spray

Meat, seafood, legumes Fats, oils Sweets Beverages

Sugar-free gelatin, popsicles, jelly, or syrup Rice-milk ice cream Water, Gatorade, sugar-free Crystal Light, sugar-free Kool-Aid, Pedialyte

Foods to avoid or limit All others If vegetables are eaten, make sure they are cooked and not raw Avoid raw fruits All others

Avoid beans Limit butter, margarine, and oils No fried foods All others Avoid carbonated beverages, soda, juices, high-sugar drinks

Table 10.2  Constipating diet phase 2 Phase 2 Food group Milk Vegetables

Fruits Starch, grains

Food recommendations All milk products allowed, but limit to 500 mL (16 oz) total per day Vegetable juice without pulp Vegetables that are well cooked Green beans, spinach, pumpkin, eggplant, potatoes without skin, asparagus, beets, carrots Applesauce, apples (without skin), banana, melon, canned fruit, fruit juice (without pulp) Bread, crackers, cereals made from refined flours Pasta or noodles made from white flours White rice, pretzels, white potatoes (without skin), dry cereal

Foods to avoid Any milk or cheese product (such as ice cream) with nuts or seeds Raw vegetables Vegetables with seeds

Fruit juice with pulp, canned pineapple, prunes, dried fruit, jam, marmalade Whole-grain or seeded breads Whole-grain pasta Brown rice, oatmeal bran cereal, whole-grain cereal (continued)

51

A patient with good surgical anatomy and hypermotility after a redo pull-through for Hirschsprung disease

Table 10.2 (Continued)  Constipating diet phase 2 Phase 2 Food group Meat, seafood, legumes Fats, oils Sweets Beverages Miscellaneous

Food recommendations

Foods to avoid

Meat, poultry, eggs, seafood Baked, broiled, or grilled are preferred cooking methods All oils, margarine, butter, mayonnaise, salad dressings Jelly, “Rice Dream” frozen desserts, sugar, marshmallows, angel food cake Water, Gatorade, sugar-free Crystal Light, sugar-free Kool-Aid, Pedialyte Salt, sugar, ground or flaked herbs and spices, vinegar, ketchup, mustard, soy sauce

Beans Fried or greasy meats, salami, cold cuts, hot dogs, meat substitutes Chunky peanut butter, nuts, seeds, coconut Anything containing nuts, coconut, whole-grains, dried fruits or jams Juice, regular soda, regular Kool-Aid or powdered drinks Popcorn, pickles, horseradish, relish, jams, preserves

We continued to increase the water-soluble fiber to the maximum dosage tolerable (4 g twice per day). At that point, the patient had achieved a good “peanut butter” consistency; however, the frequency of the stools varied between four and six stools per day.

QUESTION 10.11 What would you try to incorporate next to help with the desired goal of two to three BMs per day? Answer: We started the patient on loperamide to help with the hypermotility and the patient’s stooling patterns improved to three to four BMs per day which was acceptable to the family (Table 10.3). Table 10.3  Loperamide dosing Tablets = 2 mg Max dose 0.8 mg/kg/d

KEY LEARNING POINTS 1. Hypermotility is common post redo pull-through. It should be treated first with a constipating diet, then water-soluble fiber, then loperamide. This process should be done over 1–2 months gradually, so as not to induce constipation.

52

2. Family compliance to reporting and implementing changes was the key to success for achieving this patient’s desired bowel management goals. 3. The medications should be given at the same time every day. This is very important to a child with hypermotility to achieve a consistent stooling pattern. 4. Fiber dosings should always be split up and given throughout the day, not in a single dose. 5. Enemas can also be used to help control a fastmoving colon, help to limit soiling and improve a perianal rash.

SUGGESTED READING Levitt, M. A., Dickie, B., & Pena, A. 2010. Evaluation and treatment of the patient with Hirschsprung disease who is not doing well after a pull-through procedure. Seminars in Pediatric Surgery, 19(2), 146–153.

11

A child with Hirschsprung disease (HD) and hypomotility LINDSAY REILLY

CASE HISTORY A 4-year-old child presents with chronic constipation. In the early days of life, the child started a 4-year cycle of passing one or fewer stools per week, painful stooling, and passing a large fecal stool ball only after receiving a pediatric glycerin suppository, followed by “sandy and tarry diarrhea.” She had daily abdominal distension with vomiting and decreased oral intake with each one of these “flare ups,” which occurred approximately twice per month for 4 years. During that time, it was noted by the patient’s pediatrician that the child had a “tight” digital rectal exam and she was referred to a gastroenterologist (GI). The child was started on polyethylene glycol 3350 (17 grams twice daily) to treat the constipation. The patient had several trips to the emergency department for abdominal pain and distension, and was always discharged with the diagnosis of “functional constipation/severe anorectal incoordination.” On occasion, the child would be able to stool after a digital rectal exam in the emergency room.

QUESTION 11.1 What is the differential diagnosis? Answer: ▪▪ Functional constipation ▪▪ Internal sphincter achalasia ▪▪ Hirschsprung disease (HD) ▪▪ Anorectal malformation (ARM)

QUESTION 11.2 What are the key facts to help narrow your differential diagnoses? Answer: ▪▪ Tight sphincter, with pre anatomic anal stenosis ▪▪ Stooling with suppository/stooling after digital rectal exam These two clinical findings further narrow the differential diagnoses to internal anal sphincter achalasia and Hirschsprung disease. This is due to the high internal anal sphincter pressure that is found in both disorders. The use of a suppository or the digital rectal exam functions to overcome the tight internal anal sphincter and stool is able to pass through at that time.

Before we met her she was admitted to the hospital due to a severe episode of bilious emesis. Her evaluation included an abdominal X-ray. The X-ray revealed a large amount of stool throughout the colon and gas in a nondilated small bowel. The child underwent polyethylene glycol 3350 cleanout given via nasogastric tube, and rectal irrigations to augment stool removal. It was noted by the staff that there was a large amount of gas released when a large bore soft catheter was passed into the anus for the irrigation.

53

A child with Hirschsprung disease (HD) and hypomotility

QUESTION 11.3 Why would there be so much gas released from passing a catheter into the anus? Answer: Due to the absent rectoanal inhibitory reflex (RAIR) in HD and in internal sphincter achalasia, the internal anal sphincter squeezes tightly, trapping gas and stool in the colon. When the catheter was placed rectally, it bypassed the internal sphincter allowing the gas to escape. Due to this presentation, she was evaluated for the possibility of HD.

A biopsy was completed, but the results were still pending. A biopsy is a key piece of information in the case of refractory constipation. It determines the next course of treatment. Aganglionic bowel is incapable of receiving the impulses necessary to move stool through the colon. This process is complicated by an obstructive outlet (the internal sphincter). The result is stasis of stool and gas. Removal of the diseased bowel gives the patient the best possible anatomy and therefore, the greatest potential for normal stooling. The patient was discharged from the hospital after the cleanout and workup, with a diagnosis of constipation, while awaiting the biopsy results.

would need to be tapered. This is particularly prevalent in a patient such as this who presents at an older age. In this scenario, the patient could complete 3 months of rectal irrigations and allow the colon to shrink prior to the pull-through for a safer nontapered anastomosis (Figures 11.1 and 11.2). ▪▪ Identify the most important part of this contrast enema. – Inverse rectosigmoid ratio. Take note that it is not as obvious in this patient owing to the low level of the Hirschsprung disease. ▪▪ What is the presumed level of transition zone and how do you know this? – Presumed rectosigmoid transition zone. Presumed because the area of Hirschsprung is distal to the dilated portion. This is because the stool cannot progress through the area of aganglionated bowel and it backs up, thus dilating the colon proximal to it.

QUESTION 11.4 What type of bowel management would you start and why? Answer: Bowel management at this point for this patient should consist of irrigations. They should be utilized until HD is ruled out via biopsy results. This is a safety measure. An irrigation will bypass the sphincters and allow stool and gas to exit through the catheter. The addition of polyethylene glycol will thin the stool and allow it to be more easily irrigated out through the catheter. The use of large volume enemas should be avoided, as the liquid will just be retained in the colon. This will exacerbate the patient’s obstructive symptoms.

Figure 11.1  Contrast enema.

QUESTION 11.5 What other imaging would be helpful and why? Answer: A contrast enema can help map the transition zone. If the colon is significantly dilated, this is not ideal for a primary pull-through surgery, as the anastomosis

54

Figure 11.2  Contrast enema, post evacuation film.

Case history

The biopsy results came back 1 week later and identified prominent neural hypertrophy in the colon and rectum with no ganglion cells identified.

QUESTION 11.6 What is the diagnosis of this child’s constipation? A. Functional constipation B. Hirschsprung disease C. Henoch-Schonlein disease D. ARM Answer: Hirschsprung disease is diagnosed by this biopsy result showing aganglionic bowel (Answer B). Hypertrophic nerves in the presence of aganglionosis confirm the diagnosis.

easiest time to treat it. You can also give botulinum toxin if the patient is having trouble getting the stool to come out. Additionally, a dose of senna can help the hypomotility many patients with HD have.

For the last 4 months, the patient has been having daily soft bowel movements without any difficulty. Now the mother reports that she is having pain with stooling, straining to stool, and having decreased amount of stool. She is on the same daily dose of senna. Her X-ray is shown in Figure 11.3.

QUESTION 11.7 What is the treatment that will provide this patient the greatest opportunity for successful continence? A. Pull-through surgery to remove aganglionated bowel while preserving the integrity of the dentate line B. Botulinum toxin to relax the internal sphincter C. Long-term bowel management to regulate daily stool/soiling patterns D. All of the above Answer: A. A pull-through surgery to remove the diseased bowel is the most definitive treatment for this patient. Preservation of the dentate line and sphincters allows for successful continence in the future. Botulinum toxin to the internal anal sphincter is often an important adjunct in the management of HD in the postoperative period.

This patient completed daily rectal irrigations for 3 months. The contrast enema was repeated and showed a decrease in the dilation of the proximal colon. The primary pull-through surgery was completed. She had a proximal sigmoid colon transition zone. She returned 1 month later for an exam under anesthesia.

QUESTION 11.8 What is important to evaluate during the exam under anesthesia 1 month post–pull-though surgery? Answer: It is important to evaluate for stricture, as this is the most common time to start developing one and the

Figure 11.3  Abdominal X-ray.

QUESTION 11.9 Correlate Figure 11.3 with the clinical symptoms and provide a course of treatment. Answer: The signs and symptoms reported are indicative of outlet obstruction and most likely caused by the tight internal sphincter associated with HD not at all surprising in this age patient. Additionally, Figure 11.3 shows stool in the descending colon and neo-rectum. The sphincter will not relax and this does not allow stool to exit. There is gas retained in the stomach and a slight amount in the ascending colon. This patient needs repeat botulinum toxin to her internal sphincter. It has been 4 months since her last injection. The botulinum toxin injection lasts on average 3 months.

55

A child with Hirschsprung disease (HD) and hypomotility

Table 11.1 (Continued)  Clinical pearls of Hirschsprung’s colonic irrigations

QUESTION 11.10 Are there alternative treatments if she cannot get to the center to receive her injections right away? Answer: Yes, good quality irrigations are always an option, as they bypass the internal sphincter and allow for the escape of gas and stool. It is vitally important to have the family trained in doing these, because the stasis of stool in the colon of a patient with HD can put them at high risk for Hirschsprung associated enterocolitis.

The patient returned for anal botulinum toxin injection. With the right combination of botulinum toxin injections, daily senna, and fiber, she was able to consistently and comfortably empty her colon (Table 11.1). Table 11.1  Clinical pearls of Hirschsprung’s colonic irrigations Pearl Choose the right catheter.

Irrigate until clear.

Irrigate using small fluid volumes.

Monitor output.

Change position.

Rationale Use the largest possible flexible catheter (a foley cather is ideal) (#24 French for older than 1 year old and #20 French for younger than 1 year old). The larger diameter allows for stool to pass through the tubing. The flexibility allows for safe deep insertion and comfortable access to stool pockets in the colon. This causes less chance of retained stool. There is no set total volume for a complete irrigation. Irrigate until the fluid is nearly clear (light tea/yellow colored). Brown fluid indicates retained stool. Use small amounts (20 mL) of saline to insert repeatedly and semi-quickly to allow breakup of stool. Allow each 20 mL to drain out of catheter while moving the catheter up and down and pressing on the abdomen. Large instilled fluid volumes do not provide the pressure needed to stir up the gas and break down the stool so that it can pass through the catheter. This can leave stagnant fluid retained in the colon. Volume in = Volume out – 20 mL/kg. Retention of more than 20 mL/kg for a patient is an indication to stop the irrigation. The colon is a tube with twists and pockets. Twisting the catheter, changing body position and depth of the catheter insertion help to release all retained stool and pockets of gas. Abdominal massage is also helpful. (Continued )

56

Pearl Correlate clinically.

Soften stool.

Rationale The irrigation should release stool and gas. Evidence of successful irrigation should be decreased abdominal distension with output of gas and stool. Continued distension indicates the need for further irrigation. For frequently needed irrigations, an osmotic laxative such as polyethylene glycol 3350 can soften the stool and allow it to be more easily irrigated. Hard stool can clog the catheter.

KEY LEARNING POINTS 1. There is no substitute for a clear and well-developed history. Take the time to listen to a family and understand their course. The details in the data will often lead you to your answer. For example: Painful diarrhea, stooling with rectal exam, “tight” digital rectal exam, and large amounts of gas released when a urinary catheter is passed into the anus all point to possible Hirschsprung disease. Analysis of these key points might have led to a much earlier diagnosis. 2. Irrigations are used in HD to bypass the tight internal sphincter allowing stool and gas to escape. This helps prevent the stasis of stool and decrease the risk of enterocolitis. 3. Botulinum toxin is used to relax the internal sphincter. As it wears off, the internal sphincter will retighten and produce obstructive symptoms (painful stool passage, decreased stool, feeling urge to stool but unable, abdominal distension, gasiness).

SUGGESTED READINGS Patrus, B., Nasr, A., Langer, J. C., & Gerstle, J. T. 2011. Intrasphincteric botulinum toxin decreases the rate of hospitalization for postoperative obstructive symptoms in children with Hirschsprung disease. Journal of Pediatric Surgery, 46(1), 184–187. Tabbers, M. M., DiLorenzo, C., Berger, M. Y., Faure, C., Langendam, M. W., Nurko, S., & Benninga, M. A. 2014. Evaluation and treatment of functional constipation in infants and children: evidence-based recommendations from ESPGHAN and NASPGHAN. Journal of Pediatric Gastroenterology and Nutrition, 58(2), 258–274.

12

A patient with total colonic Hirschsprung disease and soiling STACIE LEEPER

CASE HISTORY A 16-month-old male presents to your clinic for an evaluation. He has a history of total colonic Hirschsprung disease (TCHD). His surgical history includes an ileostomy creation and colonic biopsies on the second day of life that confirmed the diagnosis of TCHD. He has no other significant medical, surgical, or family history. He takes one tablespoon of pectin two times per day to thicken the ileostomy stool. His parents report that they empty his ileostomy bag three times per day, his output is a pudding consistency, and they have been recording his output. You

QUESTION 12.1 What factors need to be taken into consideration when determining the timing of the definitive pullthrough procedure in a patient with TCHD? Answer: A patient is determined to be ready for definitive pull-through when their ileostomy output is less than 30 mL/kg/d and is thick in consistency. Thickened stool will give a child a greater chance of achieving continence, as loose stool is more difficult to control. In order to help thicken the ileostomy output, one may consider starting pectin or a water-soluble fiber. Children with an ileostomy are also at risk for failure to thrive. This is very dependent on where in the small bowel the ganglionated bowel began. It is important that a child is growing well, as adequate nutritional status is imperative for postoperative healing. In addition to monitoring a child’s growth curve, it is important to check urine sodium as sodium loss is significant in a stoma and sodium facilitates glucose transport and improves nutritional absorption.

have calculated his stool output to be 15 mL/kg/d. His parents are inquiring about the timing for his definitive pull-through procedure. It was determined that the child was a good candidate for his definitive pull-through based on the fact that he had thick output and was growing well. He went to the operating room for his ileoanal pull-through (of the ileostomy) and colectomy, via a transanal Swenson-like technique, with an abdominal incision at the ileostomy site.

QUESTION 12.2 Knowing that the perineal skin will be exposed to small bowel contents for the first time, what treatment should be considered prior to the patient leaving the operating room? Answer: Because the patient is at high-risk for skin breakdown, it is important be proactive with regards to skin care and apply a skin barrier such as Cavilon Advanced (3M) or Marathon (Medline).

QUESTION 12.3 After the pull-through, when thinking about bowel management, it is important to understand the motility of the colon. What type of motility do you anticipate this patient will exhibit? A. Hypomotility B. Normal motility C. Hypermotility Answer: Because this patient has no colon, they will need aggressive treatment for hypermotility.

57

A patient with total colonic Hirschsprung disease and soiling

After surgery, the patient stools well and is tolerating a regular diet after 6 days. He is ready for discharge.

QUESTION 12.4 Based on his anticipated hypermotility, what type of education should the parents receive prior to discharge with regard to the child’s diet? Answer: The parents should have the child follow a constipating diet (Figure 12.1). Diet is the first step in managing hypermotility. A constipating diet can provide bulk as well as help reduce the frequency of stooling. In particular, he should avoid high sugar food and drinks. Food Group

Example of Constipating Foods

Milk

All milk products allowed, but limit to 500 grams (16 oz) per day

Vegetables

Vegetable juice without pulp, wellcooked vegetables, green beans, spinach, pumpkin, eggplant, potatoes without skin, asparagus, beets, carrots

Fruits

Applesauce, apples (without skin), banana, melon

Starch, grains

Bread, crackers, cereals made from refined flours, pasta or noodles made from white flour, white rice, pretzels, white potatoes (without skin), dry cereal

Meat, seafood, legumes

Baked/broiled/grilled meats, poultry or fish, lean deli meats, eggs

Fats. oils

Very limited amounts of all oils, margarine, butter, mayonaise

Sweets

Sugar-free gelatin, popsicles, jelly or syrup, rice-milk ice cream

Beverages

Water, sugar-free Gatorade, sugar free Crystal Light, sugar-free Kool-Aid, pedialyte

Figure 12.1  Constipating diet.

QUESTION 12.5 Besides diet instructions, what other education is critical prior to discharge of this patient? A. Skin breakdown prevention/treatment B. Laxative administration C. Rectal enema teaching

58

Answer A: Because this patient is at high risk for skin breakdown, the parents should receive teaching about skin breakdown prevention and treatment seen in Figure 12.2. They should also be educated to perform frequent diaper changes.

PREVENTION OF DIAPER RASH • Clean the skin with warm water • Apply Remedy No-Rinse Foam Cleanser (Medline) • Pat dry with soft, dry cloth wipe (do not use baby wipes as they can be irritating to the skin) • After cleaning the skin, apply a protective barrier such as Cavilon No-Sting Barrier Film (3M) • Finally, apply Proshield Plus Skin Protectant (Smith & Nephew, Inc.) on the skin TREATMENT OF MILD DIAPER RASH • Clean the skin with warm water • Apply Remedy No-Rinse Foam Cleanser (Medline) • Pat dry with soft, dry cloth wipe (do not use baby wipes as they can be irritating to the skin) • After cleaning the skin, apply a protective barrier such as Cavilon No-Sting Barrier Film (3M) • Apply Critic-Aid Skin Paste (Coloplast) to the skin with each diaper change TREATMENT OF MODERATE TO SEVERE DIAPER RASH (if the skin is bleeding, wet or weepy- follow these steps) • Clean the skin with warm water • Apply Remedy No-Rinse Foam Cleanser (Medline) • Pat dry with soft, dry cloth wipe (do not use baby wipes as they can be irritating to the skin) • Apply a dusting of Stomahesive Protective Powder (ConvaTec) to clean skin • Dab the skin with a Cavilon No- Sting Barrier Film (3M) to set the powder and create a dry surface • Apply a layer of Ilex Skin Protectant Paste (Ilex Health Products) Allow the Ilex to dry for about 1 minute • Cover the Ilex with a layer of Vaseline or petroleum jelly • Wipe off any stool and petroleum jelly during each diaper change, leaving the layer of Ilex in place • Remove the Ilex once daily by soaking in a tub of warm water

Figure 12.2  Skin care instructions.

Case history

The parents call at approximately 2 weeks postoperatively reporting that their son is having four to five loose stools per day and that he is having perineal skin excoriation. They are feeding him a constipating diet and are avoiding high sugar foods. The perineum is shown in Figure 12.3.

Figure 12.4  Perineum.

QUESTION 12.7

Figure 12.3  Perineum.

QUESTION 12.6 What is your next step? Answer: The patient is already on a constipating/low sugar diet, so the next step in hypermotility management is to start some pectin or water-soluble fiber. Pectin is a naturally occurring carbohydrate that is found in fruits and is often used as a thickening agent. Water soluble fiber binds water and helps to bulk the stool. Bulking the stool will help decrease the frequency of stool, which should in turn allow the skin to heal, because perineal irritation is related to time of stool contact on the skin.

The patient returns for his 1-month postoperative check. He is following a constipating diet and taking 2 g of watersoluble fiber twice daily. He is now having approximately eight to ten loose stools per day. He is growing well and not having any episodes of abdominal distention or fever. On physical examination, you notice his perineal area to be erythematous with skin breakdown (Figure 12.4). His parents are following the skin care instructions that were provided to them at the time of hospital discharge (see Figure 12.2).

How would you proceed with this patient given that he is having frequent loose stools despite adhering to a constipating diet and taking daily water-soluble fiber? Answer: Because the patient is having greater than five liquid stools per day, the next step is to start loperamide (Imodium). Loperamide decreases the tone of the smooth muscle of the intestinal wall, which slows transit time allowing for greater water absorption. It is recommended that the pill form of loperamide be used as the liquid form contains glycerin as an inactive ingredient, which can stimulate the bowel and can cause more frequent stooling (Figure 12.5). Loperamide (Imodium) dosing guide: Tablets = 2 mg Start at 0.25 mg/kg divided BID to TID. Minimum dose 1 mg BID. Max dose 0.8 mg/kg/day or 16 mg.

Figure 12.5  Loperamide dosing.

In this case, reapplication of a skin protectant (such as Cavilon Advanced, 3M or Marathon, Medline) would be beneficial. It is also important to assess the skin for a yeast rash and treat as necessary with antifungals such as nystatin powder or oral fluconazole. 59

A patient with total colonic Hirschsprung disease and soiling

Two months later, the patient returns for a follow-up clinic visit. His current regimen is a constipating diet, twice daily water-soluble fiber and loperamide three times per day (he is taking the maximum daily dose for his weight). He continues to struggle with frequent stooling (six  to eight stools per day) throughout the day, but his perineal rash has dramatically improved. The perineum is shown in Figure 12.6.

Answer: B. Consider collaboration and consultation with GI physician who specializes in short bowel syndrome. Such providers deal with hypermotility often and can be an enormous help with these patients with Hirschsprung disease. Cholestyramine, Lomotil (Diphenoxylate + Atropine), and a clonidine patch are other treatment options that could be considered. Cholestyramine is a bile acid sequestrant and is commonly used to treat diarrhea resulting from bile acid malabsorption. Lomotil is an antidiarrheal medication that works by slowing the movement of the colon, therefore decreasing the number and frequency of bowel movements (Figure 12.7).

Step 1

Step 2

Step 3 Figure 12.6  Perineum. Step 4

QUESTION 12.8 What is the next step?

Step 5

A. Stop loperamide (Imodium) B. Start hyoscyamine (Levsin) C. Start cholestyramine Answer: The next step in the management of hypermotility is to add hyoscyamine (Levsin). Hyoscyamine is an anticholinergic that is used to slow the motility of the colon.

QUESTION 12.9

Step 6

• Constipating diet • Fiber/pectin • Loperamide (Imodium) • Start at 0.4–0.5 mg/kg, must monitor stool output carefully and adjust dose accordingly • Max dose 0.8 mg/kg/d • Hyoscyamine (Levsin) • Start 0.0625 per dose, max dose 0.75 mg total daily dose • Cholestyramine • 80 mg/kg/dose up to TID • Lomotil • Dose = 0.3–0.4 mg/kg/d divided QID, max 10 mg/d for 2–12 yo, 20 mg/d for >12 yo • Tablets contraindicated 2 mm at L5-S1) may be present, and could be the cause of neurogenic bowel. In patients with severe constipation that is refractory to medical management, this study should be considered.

QUESTION 21.3 How would you interpret these findings? Answer: The spinal MRI shows a disc bulge at L5-S1, and no tethered cord or other spinal anomalies. The patient is currently not complaining of back pain, numbness or tingling, with or without radiation to his lower extremities. The findings were considered an incidental finding. Should future symptoms occur, further follow-up would be warranted (Figures 21.1 and 21.2).

What could be the cause of the cramping, decreased appetite, and daily accidents after using MiraLAX and senna? Answer: High dose laxative use can cause significant side effects (e.g., nausea, vomiting, bloating, and abdominal pain), particularly if the colon is full when these medication are given.

QUESTION 21.2 A previous magnetic resonance imaging (MRI) had been performed. What would be the rationale for this exam? Answer: In pediatric patients with chronic constipation, an MRI can reveal abnormalities in the lumbar spine. While a

102

Figure 21.1  Lateral spinal MRI.

Case history

Figure 21.3  Contrast enema. Figure 21.2  AP spinal MRI.

With this history, we felt that he would benefit from a mechanical regimen to help him empty his colon. Treating him with medicines given the behavioral component we did not think would be successful. Due to his anal defensiveness, rectal enemas were not an option so it was decided that a laparoscopic Malone appendicostomy to provide antegrade access for flushes would be ideal. Anal botulinum toxin injection of the anal canal was an important adjunct to compensate for sphincter withholding. This had been done before, but we felt that with antegrade flushes it might now help more. A Chait tube was placed into his Malone so he could avoid needing to intermittently catheterize it. After this intervention, he had a bowel regimen consisting of 500 mL of normal saline, 30 mL of glycerin, and 27 mL of Castile via an antegrade flush. Over the next several months, he was having hard stools with very little output most days. A bowel management program was recommended to fine-tune the flush regimen to help eliminate the daily accidents. The patient comes for a bowel management program and a contrast enema is ordered prior to this visit to assess the antegrade flush (Figures 21.3 through 21.5).

Figure 21.4  Contrast enema.

Figure 21.5  Post-evacuation film.

103

A patient with severe functional constipation, fecal impaction, and soiling

QUESTION 21.4

QUESTION 21.6

How do you interpret the findings on the contrast enema done via the Malone?

Would you make any regimen changes at this time?

Answer: The contrast enema shows minimal stool in the cecum and rectum. A tube in his Malone projects over the right hemiabdomen. A normal caliber colon is noted from the cecum to the rectosigmoid with the cecum slightly larger in caliber than the rest of the colon. There is no stricture or dilation. A normal rectosigmoid is observed with peristalsis. There is no reflux into the terminal ileum. The post-evacuation image reveals no bowel distention, and the majority of the contrast has been expelled.

Answer: With stool persisting in the rectum even after the flush this morning a regimen change is done in an effort to better clean the colon. Bisacodyl 30 mL, wait 5–10 minutes, then administer 400 mL of saline will be tried. Bisacodyl use will induce contractions within the bowel; it is considered to be a stronger stimulant laxative than glycerin. The delay in administering the flush will allow the bisacodyl to work on its own before dilution by the 400 mL normal saline flush.

BOWEL MANAGEMENT DAY 2 BOWEL MANAGEMENT DAY 1 The patient arrives to the clinic for day 1 of the bowel management program to fine-tune his regimen and improve emptying. He has done his morning antegrade flush using saline 500 mL, glycerin 30 mL, and Castile 27 mL (Figure 21.6).

The patient reports now that he is having increased abdominal pain after 10 minutes of instilling his bisacodyl flush, and he had four accidents during the night (Figure 21.7).

Figure 21.7  Abdominal X-ray.

QUESTION 21.7 Figure 21.6  Abdominal X-ray.

QUESTION 21.5 How would you interpret his morning X-ray? Answer: The abdominal X-ray shows stool in the ascending colon, and an otherwise normal transverse, descending, and sigmoid colon. There is also some stool in the rectum, and there is a normal bowel gas pattern.

104

How would you interpret his day 2 morning X-ray? Answer: A Mic-key button is present over the L4-L5 disc space which was placed instead of the Chait. We felt this tube was preferable as it has a balloon device that helps to prevent leakage. There are no dilated loops or signs of obstruction. There is persistence of stool in the right colon. There is scattered stool extending from the splenic flexure to the pelvis. There is no impaction or dilation.

Bowel management day 6 (Figure 21.9)

QUESTION 21.8

QUESTION 21.10

Would you do a regimen change at this time?

What suggestions based on his report and this X-ray would you make on managing his flushes to decrease nighttime stooling?

Answer: With the persistence of stool within the colon a regimen change is warranted to achieve a cleaner colon. The patient is told to lie on his right side and instill 45 mL of bisacodyl 20 minutes prior to his 400 mL tap water flush. A switch to tap water is being tried to see if it will make any difference in the patient’s cramping symptoms, which could be due to the saline. Each child is unique, therefore varying approaches must be tried with each child until a successful bowel regimen is identified. When using tap water, it is important to watch for signs of dehydration and electrolyte imbalance, specifically hyponatremia, which is mostly relevant in young children (under age 3). Additionally, having the patient lie on his right side during the 20-minute wait time may help to dissolve the stool therein.

BOWEL MANAGEMENT DAY 5 It is mid-week of the bowel management program. The patient reports having increased abdominal pain and multiple accidents during the night (Figure 21.8).

Answer: By adjusting his first flush time to earlier in the afternoon he will have more time to empty prior to bedtime and this should decrease his bedtime accidents.

QUESTION 21.11 Would you adjust his regimen at this time? Answer: More strength to his regimen is needed to try to get him cleaned out. Bisacodyl 45 mL mixed directly with 400 mL of normal saline will be done twice today, with the goal of getting him completely emptied. A regimen change was planned for tomorrow using 400 ml of tap water and increasing bisacodyl to 60 ml once daily. Once the colon is clean again, adjustments can be made based on his daily abdominal X-ray and the hope of keeping him clean for 24 hours between enemas.

BOWEL MANAGEMENT DAY 6 (FIGURE 21.9)

Figure 21.8  Abdominal X-ray.

QUESTION 21.9 How would you interpret this abdominal X-ray? Answer: There is mild to moderate volume of stool noted throughout the colon. There is a normal distribution of bowel gas and no dilated loops.

Figure 21.9  Abdominal X-ray.

105

A patient with severe functional constipation, fecal impaction, and soiling

QUESTION 21.12 The patient is clean with minimal cramping. How would you interpret today’s X-ray compared to the prior day’s X-ray? A. Decreased stool B. Unchanged C. Increased stool Answer: The X-ray shows minimal stool in the ascending colon with no significant stool throughout the transverse, descending, sigmoid colon, or within the rectum. The X-ray has decreased stool burden from the prior day’s abdominal X-ray (Answer A). No regimen change should be made at this time. Continue with 400 mL of tap water plus 60 mL of bisacodyl once daily.

BOWEL MANAGEMENT DAY 7 (FIGURE 21.10)

Answer: The patient’s colon has accumulated stool again. The patient has a slow-moving colon (hypomotile) (Answer B). Hypomotility results in decreased contractility within the colon and results in chronic constipation.

A regimen change is done today, as well as some additional testing. We started the patient on 400 mL of normal saline and increased the bisacodyl to 90 mL daily.

QUESTION 21.14 What testing should be done to better assess motility given the difficulty you are having emptying his colon? A. Anorectal manometry (AMAN) B. Colonic manometry catheter placement via cecostomy (CMAN) C. Both A and B Answer: The patient would benefit from both an AMAN and a CMAN (Answer C). AMAN will measure the contractility in the anus and rectum to determine the presence or absence of the rectosphincteric reflex. This reflex is responsible for relaxation of the internal anal sphincter when the rectum distends allowing evacuation of stool. If abnormal, it may indicate a situation whereby botulinum toxin injection of the sphincter can improve colonic emptying. A CMAN will determine how well the muscles in the large intestine are contracting; this contraction allows stool to be expelled and can determine whether a specific segment of the colon is dysmotile.

Results of the AMAN reveal a completely normal anal sphincter. The CMAN suggests colonic inertia diffusely, throughout the entire colon. This is a rare dysmotility disorder causing slow transit of the colon and requiring surgical intervention. Options would include diverting ileostomy as an initial step and possibly total abdominal colectomy later, with a right colon to rectum anastomosis. Figure 21.10  Abdominal X-ray.

KEY LEARNING POINTS QUESTION 21.13 From the X-rays presented, how would you assess the patient’s motility? A. Normal B. Hypomotile C. Hypermotile

106

1. There are no known pathophysiological causes of functional constipation/chronic idiopathic constipation. Factors that potentiate constipation include: major life events and behavioral disorders. Additionally, children’s exposure to abuse or maltreatment may affect the clinician’s options for management of their constipation.

Suggested readings

2. Remember that each patient is unique. The approach during a bowel management program is to fine-tune a regimen that the patient and family will be most amenable to during the visit and then when back at home. Adjustments in the flush—normal saline or water, glycerin, soap or bisacodyl, etc.— all can be used to fine-tune the flush. Abdominal X-rays are performed throughout the week to verify if the regimen has been effective. If ineffective, changes in the plan, sometimes on a daily basis, will be done until the colon is successfully emptied and the patient stops having accidents. 3. The goal of the bowel management week is to get the patient clean; however, despite our bowel management protocol there are times when it does not work, and further follow-up via testing (anorectal manometry, colonic manometry) will need to be done to rule out other findings which would require more aggressive treatments. 4. AMAN looks for the presence or absence of an anal inhibitory response. If absent, this would be suggestive of Hirschsprung disease. Rectal biopsy in such a case should be performed to confirm the presence or absence of ganglion cells. If ganglion cells are found to be present this would be indicative of internal sphincter achalasia, a condition in which the internal anal sphincter is unable to relax. This condition has been shown to be successfully treated with botulinum toxin injections into the rectal sphincter, allowing relaxation of the muscle complex. 5. CMAN is a study used to determine the presence and strength of contractions via the muscles within the colon. The results of the CMAN are useful to guide treatment options such as antegrade flush options and whether a colon resection is warranted.

SUGGESTED READINGS Ahmadi, J., Azary, S., Ashjaei, B., Paragomi, P., & KhalifehSoltani, A. 2013. Intrasphincteric botulinum toxin injection in treatment of chronic idiopathic constipation in children. Iranian Journal of Pediatrics, 23(5), 574–578. Dinning, P. G., Benninga, M. A., Southwell, B. R., & Scott, S. M. 2010. Paediatric and adult colonic manometry: A tool to help unravel the pathophysiology of constipation. World Journal of Gastroenterology, 16(41), 5162–5172. Koppen, I. J. N., Lammers, L. A., Benninga, M. A., & Tabbers, M. M. 2015. Management of functional constipation in children: Therapy in practice. Paediatric Drugs, 17, 349–360. McCoy, J. A. & Beck, D. E. 2012. Surgical management of colonic inertia. Clinics in Colon and Rectal Surgery, 25(1), 20–23. Rajindrajith, S., Devanarayana, N. M., & Benninga, M. A. 2017. Association between child maltreatment and ­constipation: A school-based survey using rome III ­criteria, Journal of Pediatric Gastroenterology & Nutrition. 58(4), 486–490, April 2014.

107

22

A patient with a successful rectal enema regimen but who now is unable to tolerate rectal administration ANDREA WAGNER

CASE HISTORY A 15-year-old male with functional constipation and fecal incontinence since the age of 4 presents to your clinic. He typically has one large and hard bowel movement every 4–5 days and a small volume soiling daily. He has tried senna and lactulose regimens without success. He voids normally, has no urinary incontinence during the day or night, and has no history of urinary tract infections. He has a history of autism and is currently in the custody of a family friend. He has no past surgical history. Initial evaluation of this child revealed a normal physical exam. A contrast enema via rectum (Figure 22.1) is shown below.

QUESTION 22.1 How would you describe and interpret the findings on the contrast enema? Answer: Key findings include a dilated vertically oriented rectum and sigmoid colon extending to the hepatic flexure. There is redundancy of the sigmoid colon as well. There is subtle spiculation of the rectum suggesting smooth muscle work hypertrophy of the rectum (jejunalization). These findings, coupled with his history and exam, suggest that this child has longstanding constipation with pseudo-incontinence.

QUESTION 22.2 What type of bowel management regimen would best address this child’s soiling?

Figure 22.1  Contrast enema.

108

Answer: Two main treatment modalities, rectal enemas and oral laxatives were discussed with the child and his care provider. The family and patient felt he had already tried several oral treatment regimens without success. The decision was made to start him on a rectal enema regimen with the understanding that this treatment modality would provide the faster, more reliable and predictable route to end his soiling. During bowel management week, he was started on a bowel regimen of daily rectal enema with 500 mL of saline and 30 mL of glycerin. On day 1 of bowel management week, the patient reports good stool output following the enema, but with stool leakage overnight.

Case history

QUESTION 22.3

QUESTION 22.5

What does his X-ray reveal (Figure 22.2)?

What does his X-ray reveal (Figure 22.3)?

Answer: There is marked residual contrast on his day 1 X-ray following administration of the enema with 500 mL of saline and 30 mL of glycerin.

Answer: There is persistent residual contrast on the X-ray.

Figure 22.3  Abdominal X-ray.

QUESTION 22.6 Figure 22.2  Abdominal X-ray.

QUESTION 22.4 What is your plan for day 2 of bowel management week given this history and X-ray? Answer: Increase the volume of the rectal enema to 600 mL of saline and 30 mL of glycerin. On day 2 of bowel management week, the patient reports good stool output following the enema, but slightly less volume than on day 1. He again has stool leakage overnight.

What is your plan for day 3 of bowel management week given this history and X-ray? Answer: Increase the amount of stimulant to increase the overall strength of the enema regimen. New regimen: 600 mL of saline, 30 mL of glycerin, and 9 mL of Castile soap. On days 4–7 of bowel management week, the patient reports excellent stool output following the enema. He denies nausea, vomiting or abdominal pain. He has had no soiling in his pull ups overnight.

QUESTION 22.7 What does this X-ray from day 7 of bowel management show (Figure 22.4)? Answer: This X-ray reveals excellent stool clearance from the rectosigmoid colon. Therefore, the enema of 600 mL of saline, 30 mL of glycerin, and 9 mL of Castile soap remained his regimen at the completion of his bowel management week. You ask the patient to return in 1 month.

109

A patient with a successful rectal enema regimen but who now is unable to tolerate rectal administration

QUESTION 22.9 Would you make any changes in the short-term to his regimen given the history and X-ray findings? Answer: Because he is not adhering to the daily prescription stool may be accumulating over time. You must reinforce the importance of the daily enema administration. Increase strength to 600 mL of saline, 30 mL of glycerin, and 18 mL of Castile soap. Follow-up in 2 weeks.

QUESTION 22.10 What can you do regarding noncompliance with rectal enemas in this patient? Figure 22.4  Abdominal X-ray.

At the 1 month follow-up clinic v isit, the patient and the care provider, a maternal great aunt, describe waxing dedication to the once daily enema regimen. The patient denies fecal soiling.

QUESTION 22.8 What does his X-ray reveal (Figure 22.5)? Answer: The X-ray reveals residual stool following the enema in the rectosigmoid colon.

Answer: Talk with the patient and caregiver to best understand the reason for noncompliance. This child is able to adhere to a 30–45-minute sit time following administration of the enema. He tolerates the enema well without nausea, vomiting, or abdominal pain. He has good stool clearance clinically and radiographically. The treatment plan has eradicated his chronic soiling. Acknowledge that the rectal administration may be the most burdensome aspect of this treatment plan. This is particularly common in older children or adolescents who are appropriately trying to gain independence. In this patient, you can consider placement of a Malone appendicostomy for antegrade enema administration. The Malone appendicostomy allows the child to administer the same effective enema, and maintain privacy during flushes and independence with self-care. This patient underwent Malone appendicostomy and is doing well with daily Malone flushes.

QUESTION 22.11 For such a patient, at what point might you consider testing with anorectal and colonic manometry?

Figure 22.5  Abdominal X-ray.

110

Answer: The patient plans to do another bowel management week in the future, after 6 months of successful antegrade flushing, this time with an oral laxative regimen. Had the patient not been successful with oral laxatives or with enemas, you should consider referral for colonic and anorectal manometry testing. It is possible that he has colonic dysmotility and might benefit from a resection of dysmotile sigmoid colon. He may also have pelvic floor dysfunction and benefit from anal botulinum toxin injection or biofeedback to better coordinate his sphincter muscles.

Case history

KEY LEARNING POINTS 1. Allow older children to be active participants in their care plan. Help them understand the different treatment options, long-term plans, and reasoning behind medical decisions. 2. Rectal enemas do not have to be prescribed long term, but in some cases can be used as a trial prior to creation of Malone appendicostomy for antegrade enema administration or as a bridge until the patient is able to try or retry an oral laxative regimen. 3. After several months of successful rectal or antegrade enemas in a patient with functional constipation, you should repeat a bowel management week with laxatives if the patient and family is motivated to do so, and there are no behavioral limitations. a. The child may require a smaller laxative dose or be more successful with laxatives after compliance for a time with rectal enemas or antegrade enemas. b. If the patient does not experience success with reattempted laxatives, then such a patient should undergo further evaluation with anorectal and colonic manometry testing.

111

23

A patient with severe functional constipation who has failed laxative treatment and both rectal and antegrade enemas JULIE GERBERICK

CASE HISTORY An 8-year-old female is referred to your clinic for evaluation of her chronic constipation. She has a history of lifelong stooling problems. She currently has one bowel movement each day and complains of hard stool, abdominal pain, and straining. She has required inpatient hospitalizations twice this past year for a bowel cleanout. She takes the oral osmotic laxative polyethylene glycol 3350 (MiraLAX®) 17 g daily, the oral stimulant laxative senna 65 mg daily and a small volume over-the-counter saline rectal enema as needed. Despite these, she has fecal soiling three to four times a week. She urinates five to six times a day with complete control, but has urinary frequency, urgency, and urinary tract infections three to four times a year. She has been hospitalized once in the past year for pyelonephritis. Figure 23.1  Contrast enema completed within the last year.

QUESTION 23.1 What is polyethylene glycol 3350 (MiraLAX®)? What is senna? How do their actions differ? Answer: Polyethylene glycol 3350 is an oral osmotic laxative that makes stool soft, but does not help it come out. Senna is an oral stimulant laxative that promotes stooling, by inducing peristalsis. The starting senna dose is 1–2 mg/kg with a maximum dose based on the patient’s tolerance and symptoms.

QUESTION 23.2 What are the findings on this contrast enema? Answer: The contrast enema shows mild redundancy and dilation of the rectosigmoid.

QUESTION 23.3 What is your plan for this patient?

The child had a contrast enema in the past year, which is shown in Figure 23.1. 112

Answer: The child will benefit from a devoted bowel management program.

Case history

The X-ray in Figure 23.2 was obtained at the beginning of bowel management week.

QUESTION 23.4 What does this X-ray show with regard to stool volume? Answer: The X-ray shows stool in the left colon and some stool in the right colon.

QUESTION 23.6 What recommendations should be made from a urologic point of view? A. Importance of daily bowel movements to minimize retention of stool B. Importance of daily cleansing of the perineum and introitus with soap and water to kill off the uropathogenic bacteria that may colonize in that area C. Importance of voiding every 2–3 hours while awake D. Importance of anticholinergics for overactive bladder E. Importance of all the above Answer: Recommendations would include the importance all of the above (Answer E) to decrease urinary retention, which may contribute to her urinary symptoms and urinary tract infections.

Figure 23.2  X-ray from the first day of bowel management week.

The child had varying results during bowel management week and had X-rays that showed either an increase in stool in the colon or a clean colon while on senna which was increased to 100 mg and then decreased back to 75 mg once daily over the course of the week with 2 g of water-soluble fiber added twice daily. The child complained of abdominal cramping and nausea throughout the week. She also had days with accidents and days with no accidents. Figure 23.3 shows her X-ray at the end of bowel management week.

QUESTION 23.5 What would you do next? Answer: It was decided to start this child on a once daily dose of the oral stimulant laxative senna 75 mg and watersoluble fiber 2 g daily. This senna dose was increased from her previous dose of 65 mg as she was having stool accidents four times each week and had the X-ray findings which we concluded meant she was not emptying her colon adequately (Figure 23.2). The water-soluble fiber was added to bulk the stool in the colon to improve the sensation of stool and decrease the number of stools each day. The laxative regimen was initiated rather than a rectal enema regimen as the family expressed significant concern with the administration of rectal medications or enemas.

The child had a renal ultrasound during bowel management week which was normal.

Figure 23.3  X-ray from the last day of bowel management week.

113

A patient with severe functional constipation, failed laxative treatment and both rectal and antegrade enemas

QUESTION 23.7 What does this X-ray show with regard to stool volume? A. Stool in the right and left colon, but a clean rectum B. No stool in the colon C. Hard to tell with this image D. Stool in the transverse colon but a clean rectosigmoid

Over the course of the next several months, the child experienced decreased stool output and presented to the emergency department three times with abdominal pain, soiling, bloating, and impaction. Figures 23.4 and 23.5 show her X-ray images from one of these visits.

Answer: The X-ray shows stool in the right and left colon, but a clean rectum (Answer A).

QUESTION 23.8 What would you do next? Answer: It was determined to have the child complete the bowel management program week on the regimen of 75 mg of senna daily and 2 g of fiber twice daily to  give her body time to adjust to a consistent regimen. The child was encouraged to sit on the toilet when she had the abdominal cramping to attempt to pass stool.

Figure 23.5  X-ray from emergency department visit, view of the pelvis.

QUESTION 23.9 What do the X-rays show? What would you do? Answer: The X-ray shows moderate stool in her colon and a heavily impacted rectum. She was admitted to the hospital for a bowel cleanout and was discharged home after three days on 125 mg of senna, 68 g of the oral osmotic laxative polyethylene glycol 3350 once daily and 2 g of water-soluble fiber twice daily.

The child presents for follow-up and the family reports she is stooling one to two times per day. The stool is hard and she continues to have nausea, abdominal cramping, and daily stool accidents with leakage. The X-ray image in Figure 23.6 was obtained.

Figure 23.4  X-ray from emergency department visit.

114

Case history

QUESTION 23.10

QUESTION 23.12

What does the X-ray show?

What is your next step?

Answer: The X-ray shows a clean colon.

Answer: Owing to the patient’s ongoing clinical course with constipation requiring hospitalization, persistent accidents, abdominal pain, and several failed attempts at medical management, it was decided to obtain anorectal manometry and colonic manometry testing to evaluate for any sphincter or motility concerns, which may explain her failure of medical management.

Results of the patient’s motility testing: • Anorectal manometry: Intact rectoanal inhibitory reflex (RAIR), normal resting pressure, normal squeeze pressure, abnormal push test consistent with external anal sphincter dyssynergia • Colonic manometry: Normal with multiple and strong propagating contractions post-stimulant throughout the colon

QUESTION 23.13

Figure 23.6  X-ray from follow-up clinic visit.

QUESTION 23.11 What changes would you make with the child’s bowel regimen after reviewing this X-ray? A. Increase the senna dose and polyethylene glycol 3350 dose B. Decrease the senna dose and polyethylene glycol 3350 dose C. Make no changes D. Decrease the senna dose but increase the polyethylene glycol 3350 dose Answer: The patient has a clean colon with abdominal cramping and stool accidents. You can conclude from this that she is being overstimulated and the decision was made to decrease the senna dose to 112.5 mg once daily and polyethylene glycol 3350 dose to 17 g daily (Answer B).

With the child’s ongoing challenges with stimulant laxatives and the findings from the motility testing it was decided she would benefit from which of the following: A. Anal botulinum toxin B. Biofeedback therapy C. Rectal enema option D. Psychological support E. All of the above Answer: In collaboration with the gastroenterology/ motility team it was decided that all of the above would be most appropriate for this child (Answer E). The child will first attend a second bowel management week to try rectal enemas, to see if she improves the symptoms of abdominal and rectal pain, and this could decrease the soiling.

The child started a second bowel management program week with rectal enemas of 500 mL of saline and 30 mL of glycerin. Oral laxatives were stopped. Her X-ray to begin the week showed stool throughout the colon with rectal stool distension (Figure 23.7). 115

A patient with severe functional constipation, failed laxative treatment and both rectal and antegrade enemas

QUESTION 23.14 What would the X-ray findings suggest should be the following regimen recommendations: A. Increase glycerin B. Increase saline C. Increase Castile soap D. Leave the regimen the same Answer: Based on the X-ray with colonic and rectal stool burden, increasing the Castile soap is the correct answer (Answer C). We would not recommend increasing the saline volume or glycerin at this time. Increasing the saline volume would dilute the additives/irritants and may decrease the effectiveness of the flush. Increasing the glycerin may cause an increase in side effects such as cramping or nausea. Figure 23.7 X-ray from the first day of the second bowel management week.

Mid-bowel management week on a rectal enema regimen of 500 mL of saline, 30 mL of glycerin, and 9 mL of Castile soap, the child had no fecal soiling, no abdominal distension or pain, and the results can be seen in the X-ray image in Figure 23.8.

Figure 23.8 X-ray from the middle of the second bowel management week.

116

The child went home from the second bowel management week on a regimen of 500 mL of saline, 30 mL of glycerin, and 18 mL of Castile soap. She denied abdominal  pain, nausea, or vomiting, and had no soiling. The child’s X-ray demonstrated only a mild colonic stool burden at the end of bowel management as shown in Figure 23.9.

Figure 23.9  X-ray from the end of the second bowel management week.

Case history

Over the next six weeks, the child became defiant with rectal enemas and would not allow her mother to complete the administration. She started having trouble sleeping and began acting out and having trouble concentrating in school.

Figure 23.10 is the patient’s X-ray at the beginning of inpatient bowel management.

QUESTION 23.15 What could you offer this patient? Answer: An antegrade flush option in conjuction with anal botulinum toxin to relax the anal sphincters was scheduled as the child had resolution of her abdominal and rectal pain with the rectal enemas.

The antegrade flush option and anal botulinum toxin were performed and the child was discharged home on her previous flush regimen of 500 mL of saline, 30 mL of glycerin, and 18 mL of Castile soap. The family reported the child was emptying well on the current regimen. There were occasional hard stools and smearing one to two times weekly. Over the next months, the child had several antegrade regimen changes, oral medication changes, emergency department visits, and hospitalizations for hard stools, soiling, abdominal pain, nausea, vomiting, and rectal bleeding. The clinical team was concerned that the recommendations were not being adhered to.

QUESTION 23.16 What course of action would be recommended for this child? A. Another bowel management week with antegrade flushes B. Repeat motility testing C. Inpatient bowel management D. Stay the current course Answer: It was recommended for the child to have inpatient bowel management to help assess compliance with the regimen/recommendations and determine a more successful and tolerable flush regimen (Answer C).

Figure 23.10 X-ray from the first day of inpatient bowel management week.

QUESTION 23.17 What does the X-ray show? What would you do? Answer: The X-ray shows a significant stool burden. A mineral oil rectal enema was given prior to the patient’s flush of 600 mL of saline and 90 mL of glycerin to clean the rectum. This regimen caused significant cramping and nausea, therefore during days 2–4 of bowel management the glycerin was decreased to 60  mL and this eliminated the symptoms. Her stool output decreased over the course of several days. An exam under anesthesia found hard stool. A dose of 17 g of polyethylene glycol 3350 was added to the regimen twice daily. It became clear by the end of the third bowel management week that the patient was failing both antegrade flushes and rectal enemas and meets defecatory dysfunction criteria.

117

A patient with severe functional constipation, failed laxative treatment and both rectal and antegrade enemas

QUESTION 23.18 What would you do? Answer: The next steps include repeating colonic manometry and plan for sacral nerve stimulator (SNS) evaluation for defecatory dysfunction criteria (see Figure 23.11) (Lu, 2017; Lu et al., 2018). Referral for SNS evaluation

Clinical Preoperative Questionnaire1 Cleveland Clinic Constipation Score Fecal Incontinence Quality of Life Fecal Incontinence Severity Index Vancouver DES Score Krickenbeck Classification

Defecatory Dysfunction? Based on criteria on Slide 3

Imaging/Functional Studies Lumbar/sacral spine MRI Urodynamic studies

1 Includes diagnoses, primary symptoms,

and current therapies as listed on Slide 2

2 if history of febrile UTI, hydronephrosis

hydroureter, or anorectal malformation

Further studies based on additional criteria

Urinary Dysfunction? Based on criteria on Slide 3

Anorectal Malformation?

Hirschsprung Disease?

Renal US +/-VCUG/Cloacagram2

Rectal exam (anal location, stricture, prolapse) Pelvis MRI A/P and L sacrum XR

Exam under anesthesia (dentate line, stricture, cuff, transition zone)

Anorectal manometry Sitz marker study +/- Colonic manometry +/- Contrast enema

Defecatory dysfuction refractory to medical therapy OR Urinary dysfunction Based on criteria on Slide 3

Stage 1 SNS4

Follow up appointment 2–4 weeks after Stage 25 Postoperative Questionnaire6 Repeat all clinical scores (CCCS, FIQL, FISI, DES, Krickenbeck) Follow up appointment 3 months after Stage2 Postoperative Questionnaire Repeat all clinical scores Renal US if abnormal at baseline Urodynamic studies if abnormal at baseline

Clinical improvement Based on criteria on Slide 4

No improvement

Remove SNS Follow up appointment 6 months after Stage 2 Postoperative Questionnaire Repeat all clinical scores Anorectal manometry if abnormal at baseline Sitz marker study if abnormal at baseline

Stage 2 SNS4

Follow up appointment 9 months after Stage 2 Postoperative Questionnaire Repeat all clinical scores

4 Ambulatory encounter

OR Requirements: Stage 1 requires 90 minutes in OR with fluoroscopy and Stage 2 requires 60 minutes Anesthesia: Prone placement and no paralytic use for Stage 1 Perioperative antibiotics: Preoperative cefazolin or clindamycin if allergy Postoperative pain control: Hydrocodone if no contraindication

Follow up appointment 12 months after Stage 2 Postoperative Questionnaire Repeat all clinical scores Repeat Postoperative Questionnaire and all clinical scores every 12 months afterwards 5 Follow up may occur online or remotely if distance is prohibitive

6 Includes complications and current therapies as listed on Slide 5

Figure 23.11  Sacral nerve stimulator evaluation protocol.

118

Suggested readings

QUESTION 23.19 In the evaluation phase of the SNS protocol it is clear this patient has defecatory dysfunction. Which studies would you anticipate reviewing for this patient? Answer: The studies you may review include anorectal manometry, sitz marker study, colonic manometry (optional), and contrast enema (optional). The information from these studies will help determine what may be contributing to her failed management and determine if she is a candidate for the SNS stage 1 trial. If the patient is a candidate for the trial and experiences improvement in symptoms, the patient should progress to stage 2 for her permanent SNS placement and ongoing follow-up.

This patient will also continue to follow urology for complaints of dysuria, incontinence, and urgency. She is encouraged to complete timed voiding every 2–3 hours whether she has the sensation to void or not. She has been instructed to double void with each trip to the toilet and continue pelvic floor relaxation with biofeedback exercises. The possible placement of the SNS may also benefit this patient’s urinary symptoms by improving fecal burden.

dyssynergia found during anorectal manometry, as it may assist in the relaxation of both sphincters and allow for easier passage of stool. 5. Consider anorectal biofeedback which utilizes neuromuscular conditioning techniques to treat patients with fecal incontinence or chronic constipation associated with dyssynergic constipation. Biofeedback is a tool to help patients learn how to perform muscle exercise training to relax the external anal sphincter and allow for easier passage of stool. 6. The rectal enemas were successful in this scenario but the antegrade flush option was not successful due to her pelvic floor dyssynergia. The rectal enema catheter helps keep the anal sphincters open and allow for evacuation of stool. 7. Inpatient bowel management may be necessary to assess compliance with the regimen and need for additional patient/family education. 8. An SNS is an option for defecatory dysfunction. 9. Urinary dysfunction is not uncommon in patients with defecatory dysfunction.

SUGGESTED READINGS KEY LEARNING POINTS 1. It must be determined if the patient’s symptoms are related to constipation prior to considering any operative intervention such as an antegrade flush option. 2. If the child continues to have the same symptoms (abdominal and rectal pain) on rectal enemas, an antegrade option may not be indicated and further workup of their chronic abdominal pain is needed. 3. If the child has resolution of their abdominal and rectal pain with a successful rectal enema bowel management week, then an antegrade option would be the next course of action. 4. Botulinum toxin of the internal anal sphincter may be considered for the external anal sphincter

Lu, P. L. 2017. Sacral neuromodulation for constipation and fecal incontinence in children. Seminars in Colon and Rectal Surgery, 28, 185–188. Lu, P. L., Koppen, I. J. N., Orsagh-Yentis, D. K. et al. 2018, February. Sacral nerve stimulation for constipation and fecal incontinence in children: Long-term outcomes, pat ient benef it, a nd pa rent sat isfact ion. Neurogastroenterology & Motility, 30(2). Retrieved from https://doi.org/10.1111/nmo.13184. Wood, R. J., Yacob, D., & Levitt, M. A. 2016. Surgical options for the management of severe functional constipation in children. Current Opinion Pediatrics, 28, 370–379.

119

24

A patient who has recurrent constipation and soiling following colonic resection ANDREA WAGNER

CASE HISTORY A healthy 10-year-old female comes to your clinic with a history of functional constipation and daily fecal soiling. She passed meconium on the first day of life. She had a normal bowel movement pattern for her first 3 years. She developed constipation and soiling at age 4. She had previously tried the oral osmotic laxative Polyethylene

Figure 24.1  Contrast enema.

120

glycol 3350 and sodium phosphate rectal enemas but had continued fecal soiling. She has no other past medical history, no significant urinary history, surgical history, family history, or social history. In addition to normal anorectal physical exam findings, our initial evaluation of this patient included a contrast enema via the rectum (Figure 24.1). Post-evacuation film is shown in (Figure 24.2).

Case history

QUESTION 24.3 What is the goal of adding water-soluble fiber to this patient’s daily laxative regimen? Answer: The fiber will add bulk to her stool and make the laxative dose more effective, helping her to fully evacuate her rectum and distal sigmoid colon, preventing stool accumulation and resulting soiling.

The patient’s report of the 24 hours following medication administration and her X-ray findings are as follows. Patient report:

Figure 24.2  Past evacuation film.

QUESTION 24.1

• Three large soft bowel movements 8–12 hours after laxative dose administration • The last bowel movement was less formed than the two prior • Abdominal cramping and nausea • Decreased food intake • Soiling overnight (Figure 24.3)

How do you interpret the findings on the contrast enema? Answer: The contrast enema shows a prominent ascending colon and a redundant and dilated sigmoid colon. These findings are consistent with chronic constipation. The dilation throughout suggests a slow moving or hypomotile colon.

QUESTION 24.2 What significant finding is revealed in the post evacuation image? Answer: There is retained contrast on post-evacuation evaluation further supporting a slow moving or hypomotile colon. Figure 24.3  Abdominal X-ray.

This patient was started on a once daily dose (60 mg) of the oral stimulant laxative senna. The dose was administered once daily with the goal of producing one to two soft formed bowel movements per day. Her starting regimen was 60 mg senna once daily and 2 g of watersoluble fiber twice daily.

QUESTION 24.4 What does this X-ray show with regard to stool volume? Answer: There has been excellent stool evacuation with very little if any residual scattered stool throughout the colon. The patient has fully evacuated the contrast seen on the day prior to this X-ray.

121

A patient who has recurrent constipation and soiling following colonic resection

QUESTION 24.5

QUESTION 24.7

What changes would you make to this patient’s bowel regimen given the X-ray findings and her report of symptoms?

How would you proceed with this patient given she has failed laxatives and has documented dysmotility in her sigmoid colon?

A. Increase laxative dose B. Decrease laxative dose C. Increase fiber dose D. Decrease fiber dose

A. Malone appendicostomy for antegrade enema administration B. Resection of dysmotile sigmoid colon and Malone appendicostomy for antegrade enema administration

Answer: The decision was made to decrease the laxative dose based on the clean X-ray and patient’s report of three large bowel movements and soiling overnight (Answer B). She also had symptoms of nausea and decreased oral intake, which could indicate a too-high laxative dose.

Her regimen at completion of bowel management week was 37.5 mg senna once daily and 2 g of water-soluble fiber twice daily. The patient’s soiling was improved, but she continued to have accidents—two to three times per week.

Answer: The decision was made to proceed with Malone appendicostomy to provide patient with a trial of antegrade enemas to treat constipation and fecal soiling (Answer A). Most such patients respond well to antegrade flushes despite having a dysmotile segment of colon (usually the sigmoid). She completed another week of bowel management but despite many changes of the regimen over 3 months, was unable to find a Malone flush regimen that was effective at emptying her colon and that was well tolerated.

QUESTION 24.8 QUESTION 24.6 What is your recommendation for this patient when you find she is having daily bowel movements, but has persistent soiling at the 3 month follow-up visit? A. Colon resection B. Gastroenterology (GI) referral for colonic and anorectal manometry C. Malone appendicostomy for antegrade enema administration Answer: Because this patient has failed the trial of medical management with her having continued soiling, the decision was made to refer the patient to GI for colonic and anorectal motility studies (Answer B). ▪▪ Anorectal manometry revealed normal pressures and intact rectoanal inhibitory reflex (RAIR). ▪▪ Colonic manometry testing revealed a good response to bisacodyl and glycerin with high amplitude propagating contractions (HAPCs) throughout the colon with exception of the last 35–40 cm.

122

How would you proceed with this patient with a dysmotile sigmoid colon that has now failed oral stimulant laxatives and antegrade flushes via Malone appendicostomy? Answer: The decision was made to proceed with resection of the dysmotile and redundant sigmoid colon. Post-resection, the patient did well with Malone flushes. She had 3 months of dramatically reduced fecal soiling (one to two episodes per month). However, four months post-resection she presented with nausea and vomiting after flush administration as well as worsening fecal soiling.

QUESTION 24.9 How would you evaluate this patient with clinical failure after colonic resection? A. Abdominal X-ray B. Change the flush ingredients C. Repeat contrast study D. All of the above Answer: The decision was made to begin our investigation with an abdominal X-ray (Answer A).

Case history

Abdominal X-rays were obtained, which did not reveal a clear cause for nausea, vomiting or soiling (Figure 24.4). The flush stimulant was then changed from glycerin to Castile soap without improvement, and Castile soap to bisacodyl without improvement. The decision was then made to proceed with a contrast study via the Malone appendicostomy.

Based on the concern for an anastomotic stricture seen in Figure 24.5, the decision was made to proceed with colonoscopy. The scope could not traverse a stricture found at about 8–10 cm from the anal verge. This area was balloon dilated and the scope then passed easily. Post-dilation, the patient was resumed on her initial postresection regimen and tolerated this very well.

QUESTION 24.11 If colonoscopy in this scenario had been normal and no anatomic concern was found, what steps might you consider next to continue your evaluation and treatment of a patient with persistent constipation and soiling, post-colonic resection? A. Repeat colonic manometry testing B. Consider placement of sacral nerve stimulator Answer: Consider repeating colonic manometry testing (Answer A).

Figure 24.4  Abdominal X-ray.

QUESTION 24.10 What finding on this contrast study could explain flush intolerance and soiling after sigmoid resection and Malone placement (Figure 24.5)? Answer: The answer is possible anastomotic stricture.

KEY LEARNING POINTS 1. Abnormal colonic motility can guide surgical resection when a patient has failed oral medical management and has a redundant sigmoid on contrast enema evaluation. 2. Most patients do well with antegrade flushes despite having a dysmotile colonic segment. 3. Contrast enema evaluation post-resection can reveal anatomic problems contributing to a patient’s symptoms of persistent fecal soiling, especially if fecal soiling had initially reduced dramatically postresection. The key exam is to simulate the flush with a contrast study via the Malone. 4. If a patient is struggling post-resection with persistent fecal soiling and an anatomic abnormality is not found, other options for evaluation and treatment include repeat colonic motility testing and evaluation for sacral nerve stimulator (SNS) placement.

Figure 24.5  Contrast study.

123

25

A young adult with intractable constipation and diffuse colonic dysmotility ALESSANDRA GASIOR AND AMBER TRAUGOTT

CASE STUDY

The evaluation is performed and the results are shown in Figure 25.1.

A 28-year-old female presents with a history of constipation. Her symptoms also include bloating, abdominal pain, and the sensation of incomplete rectal emptying. She has one bowel movement every 2–3 weeks and denies any history of fecal incontinence. Her current medications include: lubiprostone (Amitiza®), linaclotide (Linzess®), polyethylene glycol 3350 (MiraLAX®), and polycarbophil (FiberCon®). In the past, she has trialed biofeedback for 2 months without any improvement. On physical exam, the anorectal exam shows normal resting and squeeze tone with normal relaxation with Valsalva.

REVIEW MULTIPLE CHOICE QUESTIONS QUESTION 25.1 What is your management plan for this patient? A. Obtain anorectal manometry B. Perform sitz marker study C. Trial biofeedback D. All of the above Answer: D. Anorectal manometry is necessary to evaluate evacuatory function, a sitz marker test is used to evaluate colonic inertia, and biofeedback is useful to assess for improvement with pelvic floor physical therapy. A 2-month period typically is an insufficient time period to assess for treatment success.

124

Figure 25.1 Abdominal film with sitz marker study on day #5 after ingestion of sitz markers.

QUESTION 25.2 What does the sitz marker study show? A. Study shows diffuse colonic intertia B. Sitz markers remaining show a normal pattern of colonic motility C. The sigmoid represents the dysmotile portion of the colon D. Further studies are needed to assess the colonic motility

Review multiple choice questions

Answer: A. More than 20% of the sitz markers remain in the colon, distributed from ascending colon to rectum, demonstrating diffuse colonic inertia.

Anorectal manometry is performed with the following results: • • • •

Normal balloon expulsion test Avg resting pressure: 53.25 mmHg Avg squeeze pressures: 135.5 mmHg Positive rectal anal inhibitory reflex (RAIR)

QUESTION 25.3 What are the relevant findings from the anorectal manometry study? A. Hirschsprung disease B. Pelvic outlet dysfunction C. Inadequate squeeze pressures D. Normal pelvic floor function Answer: D. The findings of normal balloon expulsion test rules out outlet dysfunction (also known as pelvic floor dyssynergia) and the presence of RAIR rules out Hirschsprung disease.

QUESTION 25.5 If the anorectal manometry showed an abnormal balloon expulsion (i.e., inability to push out balloon) how would this change your plan? A. Full-thickness rectal biopsy B. Laparoscopic total abdominal colectomy with end ileostomy C. Laparoscopic total abdominal colectomy with ileorectal anastomosis, no change in plan D. Pelvic floor physical therapy and biofeedback Answer: D. If the patient is unable to expel the balloon on the anorectal manometry, then the patient has pelvic floor dyssynergia. A period of pelvic floor physical therapy, to include biofeedback, would be necessary. If this fails, the operative approach would be for a laparoscopic total abdominal colectomy with end ileostomy. Ileorectal anastomosis is not advised, because the functional outlet obstruction may increase the risk for anastomotic leak in the immediate postoperative period, owing to high pressure build-up in the rectum. Patients with outlet dysfunction and an ileorectal anastomosis have frequent liquid bowel movements (often 10–20 times per day) and rectal leakage.

QUESTION 25.4

KEY LEARNING POINTS

After reviewing the sitz marker study and anorectal manometry, how would you proceed?

1. Initial evaluation should include sitz marker study to evaluate for colonic inertia. 2. Anorectal manometry should include a balloon expulsion test and evaluation for RAIR. 3. If RAIR is positive, this would rule out Hirschsprung disease. 4. The balloon expulsion test on the anorectal manometry demonstrates a negative workup for pelvic floor dyssynergia. 5. If the patient does not have pelvic floor dyssynergia, then a total abdominal colectomy with ileorectal anastomosis is the best surgical option. 6. If the patient has pelvic floor dyssynergia, a pelvic anastomosis would be ill-advised. Pelvic floor physical therapy and biofeedback should be performed prior to any operative intervention.

A. Full-thickness rectal biopsy B. Laparoscopic end ileostomy C. Laparoscopic total abdominal colectomy with ileorectal anastomosis D. Continue medical management for constipation Answer: C. A laparoscopic total abdominal colectomy with ileorectal anastomosis is the most appropriate surgical procedure. An end ileostomy is more appropriate if the patient has significant pelvic floor dyssynergia. A full-thickness rectal biopsy would only be useful to rule out Hirschsprung disease, if the RAIR was negative on the anorectal manometry, and this diagnosis was suggested by the clinical history.

125

26

A young adult with pelvic floor dyssynergia ALESSANDRA GASIOR AND AMBER TRAUGOTT

CASE STUDY A 30-year-old female presents with a history of chronic constipation. She also complains of rectal pain with defecation. She has a history of multiple episodes of hospitalizations requiring disimpaction. She has progressive bloating worsened by laxatives. Her frequency of bowel movements are only one every few weeks, and to initiate them she must sit for long periods with multiple position adjustments and digital maneuvers to evacuate her rectum. Her current medications include: 8–10 laxatives per day (senna 86 mg) as well as linaclotide (Linzess®). On physical exam, she has an anorectal exam with a hypertonic sphincter and absent relaxation with Valsalva.

QUESTION 26.1 What is the next step in the management of this patient? A. Obtain anorectal manometry B. Trial pelvic physical therapy and biofeedback C. Order sitz marker study D. All of the above Answer: D. All of the above tests are necessary to fully evaluate the patient.

EVALUATION The sitz marker study results are shown in Figure 26.1. Sitz Marker Study with results shown after day 5 of ingestion of the sitz markers. 126

Figure 26.1  Abdominal X-ray with sitz markers.

QUESTION 26.2 What does the sitz marker study demonstrate after 5 days? A. Diffuse colonic inertia B. Pelvic outlet disorder C. Segmental slow transit in descending colon only D. Normal colonic motility Answer: B. The sitz markers are all in the sigmoid colon and rectum and this coupled with a prolonged colonic transit time, demonstrating pelvic outlet dysfunction, or pelvic floor dyssynergia.

Suggested readings

Anorectal manometry: • • • •

Failure to pass balloon expulsion test Avg resting pressure: 80.1 mmHg Avg squeeze pressures: 148.1 mmHg Positive Rectal Anal Inhibitory Reflex

QUESTION 26.3 What do the anorectal manometry results tell you? A. Diagnostic of Hirschsprung disease B. Failure to pass the balloon expulsion test demonstrates pelvic floor dyssynergia C. Normal anorectal manometry D. Primary sphincter weakness Answer: B. This anorectal manometry is abnormal as it shows failure to pass the balloon expulsion test, which is indicative for pelvic floor disorder. Hirschsprung disease can be ruled out as the rectoanal inhibitory reflex (RAIR) is positive. The sphincter pressures do not demonstrate weakness.

QUESTION 26.4 After reviewing the sitz marker study and the anorectal manometry findings, how would you proceed? A. Full-thickness rectal biopsy B. Refer patient to pelvic floor physical therapy and biofeedback C. Laparoscopic loop colostomy D. Laparoscopic total abdominal colectomy with ileorectal anastomosis Answer: B. The sitz marker study does not demonstrate colonic inertia, but supports outlet dysfunction with the retained markers all within the rectosigmoid. The anorectal manometry also demonstrates pelvic floor dyssynergia. Therefore, the next step would be to refer this patient for pelvic floor physical therapy and biofeedback. Pelvic floor physical therapy serial treatments provides education on pelvic floor exercises to increase flexibility, stretch tight muscles, and strengthen weak ones. Biofeedback is a technique that can help patients see how their pelvic floor muscles are working.

QUESTION 26.5 If the patient fails to improve with pelvic floor physical therapy and biofeedback, what would you do next? A. Total colectomy with end ileostomy B. Loop colostomy without resection C. A or B D. Total abdominal colectomy with ileorectal anastomosis Answer: C. If the patient fails pelvic floor physical therapy, the next best options for surgical management would be total colectomy with end ileostomy or loop colostomy without resection. A loop colostomy bypasses the functional pelvic floor obstruction, without the morbidity of an end ileostomy for long-term dehydration or electrolyte disturbances. Loop colostomy is only appropriate if colonic motility is relatively preserved.

KEY LEARNING POINTS 1. Based on her symptoms, physical exam findings, and evaluation, this patient has pelvic floor dyssynergia. 2. Colonic transit is considered abnormal if more than 5 sitz markers (20% retained) are present on plain abdominal film after 5 days. 3. Balloon expulsion test has an 80%–90% specificity, and 50% sensitivity for dyssnergia (Rao 2008). 4. Pelvic floor physical therapy with biofeedback has been shown to be superior to diazepam or placebo in treating pelvic floor dyssynergia (Heymen et al. 2007).

SUGGESTED READINGS Heymen, S., Scarlett, Y., Jones, K. et  al. 2007 April. Randomized, controlled trial shows biofeedback to be superior to alternative treatments for patients with pelvic floor dyssynergia-type constipation. Diseases Colon Rectum, 50(4), 428–441. Rao, S. S. 2008 September. Dyssynergic defecation and biofeedback therapy. Gastroenterology Clinics North America, 37(3), 569–586.

127

27

A patient with severe constipation and a behavioral disorder KATRINA HALL, CHARAE KEYS, AND ROSE LUCEY SCHROEDL

CASE HISTORY A 10-year-old child with functional constipation manifesting as soiling, bloating, and impaction beginning at the age of 2 presents for help with difficulty stooling. Many attempts at laxative treatments had failed in the past. The patient was previously diagnosed with a mood disorder, sleep disturbance, Attention Deficit Hyperactivity Disorder (ADHD), Autism Spectrum Disorder, and disruptive behavior disorder. He has a history of atypical behaviors, including self-injury, chronic suicidal ideations, and hallucinations, and has been prescribed various medications to manage ADHD symptoms, stabilize his mood, and aid with sleep. The family disclosed a history of chronic physical and mental health conditions, in addition to barriers to resources and access to healthcare.

QUESTION 27.1 How would you manage this patient based on the social and medical information? A. Do not consider the psychosocial aspects of the case in your plan of care. B. Engage your psychosocial team members including psychology, child life, and social work at the start of the process. C. Formulate a medical plan of care and engage your psychosocial team at the next follow –up visit. D. Have the medical team make all assessments and consult psychology, child life, and social work only if they cannot solve the issue themselves.

128

Answer: B. Engaging the psychosocial team at time of initial evaluation will provide the medical team with psychosocial history that will benefit the multidisciplinary team regarding the patient’s behavior, ability to complete the recommended regimen, assess for financial barriers to care, ability to cope with the regimen, and assess the parents’ understanding of the medical recommendations. In an ideal situation, all team members meet the patient at the time of initial evaluation and then each team member utilizes their skills to provide the best care.

When the psychosocial team is utilized, they are able to advocate for the needs of patients and families, and influence decisions regarding the patient and family’s medical needs, including their goals for treatment. Through their role and specific assessments, the psychosocial team is able to define what is achievable and realistic for families given their current regimen, the recommended regimen, and their environment outside of the hospital. This child attended a week-long bowel management program and was started on a rectal enema regimen of 500 mL of saline and 30 mL of glycerin. A mechanical way to empty the colon was chosen because of prior failures with laxatives. During the initial visit, a behavior plan was created with the family for enema administration, including establishment of a routine, distraction, and contingency management with rewards and consequences for compliance. This plan encouraged the family to administer the enema at the same time each day as able, let the patient choose an activity of interest to engage in during the catheter insertion, and during the flush administration, and then allow the patient to identify high-saliency rewards for success, such as a treat or extra videogame time.

Case history

At the end of the week, the patient’s X-ray showed that he was clean when enemas were successfully administered. However, uncooperative behav ior required his caregivers to restrain him during enema administration.

QUESTION 27.2 What aspects of this child’s history may have influenced his behavioral response to the enema administration? A. Family history of chronic illness B. Social history of abuse C. Behavioral diagnoses D. All of the above Answer: D. Family history, behavioral diagnoses, and concerns regarding abuse/neglect should be discussed with the interdisciplinary team when considering a patient’s regimen. It is also imperative to discuss with parents and guardians their ability to complete the enema administration, as they may have psychosocial history that prevents them from participating in their child’s regimen.

QUESTION 27.3 After a child with behavioral problems and severe constipation has failed oral medication management and a trial of rectal enemas, what is your next course of action? A. Surgical resection B. Consider antegrade enemas C. Inpatient bowel management Answer: B. An antegrade enema regimen can promote independence and autonomy as the child can independently place the catheter in the channel created and administer the flush. The antegrade enema option can decrease the uncomfortable feeling many children complain of with rectal enema administration. The mechanical route is reliable and allows for “scheduling” of the daily stool, without concern for effects of laxatives such as nausea, cramping, or unexpected stooling.

QUESTION 27.4 What is this patient’s best option for antegrade enema administration? A. Cecostomy B. Malone appendicostomy C. Neomalone Answer: A.

This is a challenging decision with multiple factors to consider. In this patient, the medical team chose to do a laparoscopic-assisted cecostomy for a few reasons: 1. Sometimes a thick abdominal wall can present

difficulties for the surgeon where the native appendix cannot reach the umbilicus. 2. Some patients with behavioral and developmental needs may have difficulty with the sensory aspect and compliance of catheterization of the Malone site in the umbilicus. The ability to connect to an indwelling device can be less invasive for patients and promote cooperation. Note: A Malone could also be created, and a device placed in it, so that the patient has both options, catheterization or leaving in an indwelling device. After the patient attended the bowel management week, the medical team and family decided that an antegrade enema regimen would be the best option for the patient to aid with compliance. The child was admitted a day prior to surgery to receive a preoperative bowel preparation with GoLYTELY through a nasogastric (NG) tube and intravenous (IV) fluids. The reason for this preoperative bowel preparation was because the patient was not on a reliable bowel regimen and he was regularly impacted. The multidisciplinary team determined a secure device would be a better option (instead of cathing), due to his behavioral and social history. Accessing the device is often less invasive for the patient, thus decreasing the risk of triggering trauma symptoms (i.e., defiance, aggressive behaviors). “Given that there is a direct preoperative anxiety to postoperative pain correlation for typically developing children, we must also consider the potential that heightened anxiety and maladaptive behaviors could produce a more challenging and painful post-operative experience for children with Autism Spectrum Disorder. It is the healthcare 129

A patient with severe constipation and a behavioral disorder

professional’s responsibility to consider all issues when developing a plan of care for this patient population.” (Wittling et al., 2018). Antegrade flushes went well at first but two months later, the child came to the emergency department with a stool impaction.

QUESTION 27.5 What would you do first to relieve the child’s stool impaction? A. Prescribed flush through the Chait B. GoLYTELY drip via cecostomy C. Mineral oil enema D. Manual disimpaction E. Oral laxatives Answer: B. GoLYTELY was started via the cecostomy; however, the child remained impacted. Then the psychosocial team influenced the decision to take the patient to the operating room (OR) for disimpaction rather than receiving enemas.

After OR disimpaction, and continued GoLYTELY over several days, the colon was clean. After restarting a diet and daily flushes via the cecostomy, over the next few weeks he again became impacted. This occurred despite excellent compliance to the daily flush and increasing the flush concentration and volume (Figure 27.1). The team felt a motility evaluation was needed due to the failure of medical management with laxatives, enemas, and antegrade flushes. The anorectal manometry (AMAN) was normal, demonstrating no sphincter dysfunction. A colonic manometry study (CMAN) showed that the patient’s entire colon lacked the normal contractions to pass stool, which is also known as diffuse colonic dysmotility. At this point it was decided to resect the majority of his dysmotile colon and leave him with only the right colon and access for a daily antegrade enema (his cecostomy). This was achieved via a subtotal colectomy with right colon to rectum anastomosis. The cecostomy remained in the right lower quadrant. In doing this, the child maintained antegrade access to flush the right colon and rectum. This approach gives the patient a chance to maintain a more normal “bowel movement pattern.” Postoperatively, the patient returned to the inpatient unit with a peripherally inserted central catheter (PICC) Foley, epidural for pain management, and nasogastric tube to suction. The patient’s bowel function returned after 1 week. The patient’s nil per os (NPO) status interrupted a normal routine, as the patient was unable to take their behavioral medications by mouth. The patient was in the hospital for a total of 22 days.

QUESTION 27.6 What would you do if a patient is unable to take behavioral medications by mouth? A. Discontinue all behavioral health medications B. Clonidine patch to assist with behavioral management C. Consult psychology D. Provide clonidine patch and psychology consult Answer: B.

Figure 27.1  Patient’s X-ray with stool burden.

130

Interdisciplinary collaboration is essential for patients on a behavioral medication regimen to promote a smooth transition post operatively. Collaboration between the medical team, psychology, and pharmacology can promote better outcomes for the patient and reduce interruptions in the

Suggested reading

patient’s routine. Since this patient was NPO, it was important to find ways to maintain the medical therapy similar to the patient’s home medication regimen. The clonidine patch was a temporary option for the patient until he was able to start his usual behavioral medications by mouth. Leading up to the final days of this patient’s admission, the child became uncooperative with daily tasks. The patient displayed combative behavior towards the staff. A code was called by the staff due to the patient becoming combative after his mother left his bedside. The crisis team responded to the call and assisted staff in deescalating the patient.

QUESTION 27.7 What psychosocial team members should you consult now that this child and family are ready for discharge? A. Nothing, up his medication B. Psychology consult C. Child life D. Social work E. B, C, and D Answer: E. During the code, a child life specialist was present and able to redirect the child back to their room utilizing the child’s interest in videogames. A behavior management plan was created to complete the required tasks with the child’s interests being the reward for compliance with changing clothes, taking medications, and returning to his room. The social worker met with the family at his bedside to assess the needs and stressors related to ongoing admission, and also to provide counseling. Social work provided counseling regarding the family’s ongoing stressors including the mother’s own legal issues, health concerns, and overall impact of the hospitalization on the family unit.

Psychology was consulted on the day of discharge and developed a behavior plan to promote compliance with medication and oral intake. Consults to child life, psychology, and social work would ideally be made upon admission to the hospital when behavioral and familial concerns are identified. Interventions that allow the patient and family to have individualized care plans promote positive coping for the family unit, (Wittling et al., 2018). This child continues to be seen in clinic for follow-up visits regarding their cecostomy flush regimen. Psychology continues to meet with the child, as his baseline behavior had not returned to how it was prior to hospitalizations, the

explanation for which is thought to be related to medication titration. The child’s last X-ray was shown to be clean, yet he was still having occasional accidents.

QUESTION 27.8 How would you manage this patient at this time? A. Add laxatives to regimen B. Add fiber to regimen C. Add bisacodyl to flush regimen D. Change flush to twice a day Answer: B. Adding fiber to the patient’s antegrade enema regimen will help bulk the stool. A child with only a small remaining segment of colon may experience liquid stool. Water-soluble fiber given throughout the day can add bulk to their stool, slowing overall motility, and decreasing occasional episodes of soiling.

KEY LEARNING POINTS 1. Antegrade enema options should be considered for patients with history of abuse and behavioral disorders as this option promotes independence and autonomy and avoid the rectal route. 2. A patient and family psychosocial history including past history of child abuse/neglect should be taken into consideration when prescribing a medical regimen. 3. Early involvement of the psychosocial team including child life, psychology, and social work can help patients, caregivers, and families address behavioral and psychiatric concerns to promote positive coping for patients and improve overall treatment response. 4. Patients with underlying behavioral concerns benefit from continuity of medication and behavioral interventions in the context of medical treatment to maintain gains of past behavioral health treatments, as well as aid with the recovery process.

SUGGESTED READING Wittling, K., Dufur, J. P., McClain, A., & Gettis, M. 2018. Behavioral coping plans: One inter-professional team’s approach to patient-centered care. Journal of Pediatric Nursing, 41, 135–139. 131

28

A young adult with incontinence after a low anterior resection ALICIA FINN, SCOTT LAKE, AND AMBER TRAUGOTT

CASE HISTORY A 40-year-old female presents with fecal incontinence. She underwent neoadjuvant chemoradiation followed by low anterior resection for rectal cancer 5 years ago. She reports her incontinence started after surgery and has worsened since that time. She does not always feel the urge to defecate, and wears adult briefs daily both for passive stool leakage and for stool accidents. She has about four bowel movements per day, which are usually about an hour apart. They are sometimes loose in consistency. She has limited her activities outside the home, and carries extra clothes when she must go out in public. Her obstetric history includes two vaginal deliveries, which were only significant for an episiotomy with the second delivery. Physical exam revealed very mild perianal skin thickening related to her prior radiation. Resting and

QUESTION 28.1 Which of the following instruments has been validated for evaluating fecal incontinence in adults? A. Cleveland Clinic Incontinence Score (CCIS, also known as Wexner score) B. Fecal Incontinence Quality of Life Scale (FIQOL) C. Gastrointestinal Quality of Life Index (GIQLI) D. Fecal Incontinence Severity Index (FISI) E. All the above Answer: E. Each of these instruments has been validated in adults with fecal incontinence. Selecting an appropriate instrument will depend on the context of the patient’s symptoms, comorbidities, and type of treatment.

132

squeeze tone were decreased on digital rectal examination. No mass or stricture was felt. This patient had a Cleveland Clinic Incontinence Score of 15, indicating moderate to severe symptoms. Her FIQOL testing indicated her symptoms severely impacted the domains of lifestyle, coping, and embarrassment, while not correlating with symptoms of depression.

QUESTION 28.2 Which of the following diagnostic studies would not be indicated for her symptoms at this time? A. Colonoscopy B. CT of the abdomen and pelvis C. Endorectal ultrasound D. Anorectal manometry Answer: B. Colonoscopy should be done to evaluate for pathologies that may contribute to her loose stools or incontinence. Endorectal ultrasound will evaluate for sphincter injury related to prior episiotomy. Anorectal manometry will evaluate defecatory mechanics, rectal sensitivity, and rectal compliance, any or all of which could be altered by prior rectal surgery and radiation. CT of the abdomen and pelvis would not be indicated.

QUESTION 28.3 What treatment recommendations can be made at the initial visit? A. Increase fiber consumption B. Trial pelvic floor physical therapy targeting sphincter strength and rectal sensation

Treatment

C. Loperamide as needed for diarrhea D. All of the above Answer: D. Fiber bulks the stool, which can help to reduce diarrhea and to improve urge sensation and rectal emptying. Antidiarrheals such as loperamide can also help with loose stools. Pelvic floor physical therapy can improve neuromuscular function.

EVALUATION

ENDOANAL ULTRASOUND QUESTION 28.5 What does the endorectal ultrasound (EUS) show (Figure 28.1)? A. Internal anal sphincter defect B. External anal sphincter defect C. Combined internal and external anal sphincter defect D. No sphincter defect Answer: D. The EUS shows that the internal and external sphincter muscles are both intact, though thin and atrophic.

Colonoscopy was performed and showed mild radiationrelated changes in the remaining rectum and a healthy appearing colorectal anastomosis at 5 cm above the anal verge. The results were otherwise normal.

ANORECTAL MANOMETRY Resting sphincter pressure (mmHg):